materi-osn-matematikasma

74
MATERI PENGANTAR OLIMPIADE SAINS NASIONAL BIDANG MATEMATIKA SMA DISUSUN OLEH: TIM PEMBINA OLIMPIADE MATEMATIKA TIM OLIMPIADE MATEMATIKA INDONESIA Juli 2009

Upload: fandy-ahmad

Post on 02-Jul-2015

4.341 views

Category:

Documents


9 download

TRANSCRIPT

Page 1: Materi-OSN-MatematikaSMA

MATERI PENGANTAR

OLIMPIADE SAINS NASIONAL

BIDANG MATEMATIKA SMA

DISUSUN OLEH:

TIM PEMBINA OLIMPIADE MATEMATIKA

TIM OLIMPIADE MATEMATIKA INDONESIA Juli 2009

Page 2: Materi-OSN-MatematikaSMA

KATA PENGANTAR

Olimpiade Sains Nasional (OSN) merupakan kegiatan yang dilaksanakan

secara rutin setiap tahun. Selain sebagai ajang kejuaraan nasional, OSN juga

bertujuan untuk mencari calon peserta yang akan dibina dan berlaga di olimpiade

sains tingkat internasional. Untuk memberikan arah pembinaan bagi peserta OSN,

terutama peserta bidang matematika SMA, maka disusunlah materi ini.

Materi yang kami susun ini merupakan draf awal yang kami kembangkan

dari silabus OSN Bidang Matematika SMA yang sudah beredar. Draf ini jauh dari

lengkap seperti yang tercantum dalam silabus. Oleh karena itu para peserta OSN

dan para pembina masih perlu mencari pelengkapnya dari berbagai sumber.

Terakhir, kami berharap bahwa dengan adanya draf ini persiapan ke OSN

tahun ini bisa lebih baik dibanding tahun-tahun sebelumnya. Pada akhirnya akan

berdampak kepada kualitas calon peserta IMO tahun depan.

Jakarta, 1 Juli 2009

Tim Pembina Olimpiade Matematika

Page 3: Materi-OSN-MatematikaSMA

DAFTAR ISI

Aljabar 1

Teori Bilangan 14

Geometri 36

Kombinatorika 60

Page 4: Materi-OSN-MatematikaSMA

ALJABAR

Hery SusantoJurusan Matematika FMIPA UM

Jl. Surabaya 6 Malang 65145Email: [email protected]

Aljabar merupakan salah satu materi pokok dalam Olimpiade Matematika Internasional (IMO),disamping geometri, ilmu bilangan, dan kombinatorik. Oleh karena itu, aljabar menjadi salah satumateri wajib di Olimpiade Sains Nasional (OSN) Bidang Matematika SMA. Para peserta OSNBidang Matematika SMA sudah mendapatkan materi aljabar sejak masih duduk di Sekolah Dasar.Menurut pengamatan penulis, sampai dengan di tingkat SMA para peserta sudah cukup banyakmendapat materi yang berkaitan dengan aljabar. Yang kurang adalah berlatih menerapkan teoriyang telah didapat itu dalam soal-soal yang berbentuk pemecahan masalah. Di sini penulis men-coba memilihkan materi-materi aljabar yang kira-kira mengarah ke materi olimpiade matematika.Penulis tidak banyak menyajikan materi yang berkaitan dengan teori tetapi lebih banyak denganpendekatan strategi pemecahan masalah.

Materi yang dibahas di sini hanya sebagian dari materi yang ada pada silabus OSN BidangMatematika SMA. Kami berharap peserta merujuk kepada silabus OSN yang sudah beredar. Dibagian akhir diberikan beberapa soal latihan dan diharapkan dapat dipakai untuk memacu dalammengembangkan kemampuan pemecahan masalah.

Pada akhirnya penulis berharap agar para peserta OSN Bidang Matematika SMA dapat meru-juk materi yang tercantum dalam silabus OSN dan membaca buku-buku tentang pemecahanmasalah untuk mengembangkan kemampuan pemecahan masalah ke tingkat yang lebih tinggi.

1 Sistem Bilangan Real

Misalkan N, Z, Q, dan R berturut-turut menyatakan himpunan bilangan asli, himpunan bilanganbulat, himpunan bilangan rasional, dan himpunan bilangan real. Masing-masing himpunan inidilengkapi dengan operasi tambah dan operasi kali disebut sistem bilangan. Sistem ini biasa di-tulis notasi himpunan beserta operasinya. Sebagai contoh sistem bilangan real ditulis (R,+,×).Selanjutnya cukup ditulis notasi himpunannya saja, yaitu R.

Berikut akan dibahas dua aksioma yang berkaitan dengan sistem bilangan real, yaitu aksiomalapangan dan aksioma urutan.

1.1 Aksioma Lapangan

1. Sifat asosiatif, yaitu untuk setiap a, b, c di R berlaku

(a) (a + b) + c = a + (b + c),

(b) (ab)c = a(bc).

2. Sifat komutatif, yaitu untuk setiap a, b, c di R berlaku

1

Page 5: Materi-OSN-MatematikaSMA

(a) a + b = b + a,

(b) ab = ba.

3. Eksistensi unsur identitas, yaitu

(a) Terdapat 0 di R yang memenuhia + 0 = a

untuk semua a di R.

(b) Terdapat 1 di R dan 1 6= 0 yang memenuhi

a1 = a

untuk semua a di R.

4. Eksistensi unsur invers, yaitu

(a) Untuk masing-masing a di R terdapat −a di R yang memenuhi

a + (−a) = 0.

(b) Untuk masing-masing a di R yang tidak nol terdapat a−1 di R yang memenuhi

aa−1 = 1.

5. Sifat distributif, yaitu untuk setiap a, b, c di R berlaku

a(b + c) = ab + ac.

Sebagai kosekuensi dari sifat-sifat pada aksioma lapangan di atas, diperoleh sifat-sifat berikut(yang dapat dianggap sebagai teorema).

1. −a dan a−1 yang memenuhi sifat 4 di atas adalah tunggal.

2. 0a = 0, untuk setiap a di R.

3. (−1)a = −a, untuk setiap a di R.

4. −(−a) = a, untuk setiap a di R.

5. (−a)(−b) = ab, untuk setiap a di R.

6.(a−1

)−1 = a, untuk setiap a di R yang tidak nol.

Sebagai suatu teorema, sifat-sifat di atas harus dibuktikan. Berikut diberikan beberapa contohpembuktian.

2

Page 6: Materi-OSN-MatematikaSMA

Bukti untuk 2. Ambil a ∈ R sebarang. Dengan menggunakan sifat identitas, invers, asosiatif,dan distributif diperoleh

0a = 0 + 0a

= (−0a + 0a) + 0a

= −0a + (0a + 0a)= −0a + (0 + 0)a= −0a + 0a

= 0.

Karena a ∈ R sebarang, maka 0a = 0, untuk setiap a di R.

Bukti untuk 3. Ambil a ∈ R sebarang. Dengan menggunakan sifat identitas, invers, asosiatif,distributif, dan sifat 2 di atas diperoleh

(−1)a = 0 + (−1)a= (−a + a) + (−1)a= −a + (a + (−1)a)= −a + (1a + (−1)a)= −a + (1 + (−1))a= −a + 0a

= −a + 0= −a.

Karena a ∈ R sebarang, maka (−1)a = −a, untuk setiap a di R.

Bukti keempat sifat yang lain digunakan sebagai latihan.

1.2 Aksioma Urutan

Terdapat himpunan P yang merupakan himpunan bagian dari himpunan bilangan real yang memenuhitiga sifat berikut.

(1) Untuk bilangan real x sebarang, berlaku salah satu dari: (i) x = 0, atau (ii) x ∈ P , atau (iii)−x ∈ P .

(2) Jika x, y ∈ P , maka x + y ∈ P .

(3) Jika x, y ∈ P , maka xy ∈ P .

Sifat (1), (2), dan (3) di atas berturut-turut disebut sifat trikotomi, sifat ketertutupan operasitambah, dan sifat ketertutupan operasi kali. Himpunan P di atas disebut himpunan bilangan realpositif. Berikutnya ada kesepakatan bahwa notasi x > 0 digunakan jika x ∈ P . Dengan demikianketiga sifat di atas dapat ditulis ulang sebagai berikut.

(1’) Untuk bilangan real x sebarang, berlaku salah satu dari: (i) x = 0, atau (ii) x > 0, atau (iii)−x > 0.

3

Page 7: Materi-OSN-MatematikaSMA

(2’) Jika x > 0 dan y > 0, maka x + y > 0.

(3’) Jika x > 0 dan y > 0, maka xy > 0.

Berikut didefinisikan relasi ”lebih besar dari” dan relasi ”lebih kecil dari” untuk dua bilanganreal. x dikatakan lebih besar dari y, dinotasikan x > y, jika x− y > 0. x dikatakan lebih kecil dariy, dinotasikan x < y, jika y − x > 0. Dapat ditunjukkan bahwa x > y ekivalen dengan y < x.Notasi x ≥ y, dibaca x lebih besar dari atau sama dengan y, digunakan jika x > y atau x = y.

Konsekuensi dari sifat-sifat di atas, diperoleh sifat-sifat berikut (yang dapat dianggap sebagaiteorema).

1. Untuk setiap pasang bilangan real a dan b pasti berlaku salah satu dari a < b, atau a = b,atau a > b.

2. Jika a < b dan b < c, maka a < c.

3. Jika a < b, maka a + c < b + c.

4. Jika a < b dan c > 0, maka ac < bc.

5. Jika a > 0 dan b > 0, maka ab > 0.

6. Jika a < b dan c < 0, maka ac > bc.

7. Untuk setiap a di R berlaku a2 ≥ 0. Selanjutnya, a2 = 0 jika dan hanya jika a = 0.

8. Jika a > b > 0, maka 1a < 1

b .

9. Jika a > b > 0 dan c > d > 0, maka ac > bd dan ad > b

c .

10. Jika a > b, maka an > bn untuk n bilangan asli ganjil.

11. Jika a > b > 0, maka an > bn untuk n bilangan asli.

Bukti sifat-sifat di atas digunakan sebagai latihan.

2 Ketaksamaan

Sifat-sifat yang berkaitan dengan aksioma urutan banyak digunakan pada masalah-masalah yangberkaitan dengan ketaksamaan. Pembahasan ketaksamaan berikut menggunakan pendekatan pe-mecahan masalah dengan beberapa contoh.

Contoh 1 Tunjukkan bahwa a + 1a ≥ 2 untuk setiap bilangan real a > 0, dan akan merupakan

kesamaan jika dan hanya jika a = 1.

Penyelesaian: Untuk setiap bilangan real a berlaku

a2 − 2a + 1 = (a− 1)2 ≥ 0.

4

Page 8: Materi-OSN-MatematikaSMA

Sehingga a2 + 1 ≥ 2a. Karena a > 0 maka

a +1a≥ 2.

Selanjutnya

a +1a

= 2⇔ a2 + 1 = 2a⇔ a2 − 2a + 1 = 0⇔ (a− 1)2 = 0⇔ a = 1.

Contoh 2 Misalkan a, b, dan c adalah bilangan real positif dan a + b + c = 1. Tunjukkan bahwa

1a

+1b

+1c≥ 9.

Penyelesaian: Dari a + b + c = 1 diperoleh

1a

= 1 +b

a+

c

a1b

=a

b+ 1 +

c

b1c

=a

c+

b

c+ 1

Sehingga

1a

+1b

+1c

=(

1 +b

a+

c

a

)+

(a

b+ 1 +

c

b

)+

(a

c+

b

c+ 1

)=

(a

b+

b

a

)+

(a

c+

c

a

)+

(c

b+

b

c

)+ 3

Menurut Contoh 1, ab + b

a ≥ 2, ac + c

a ≥ 2, cb + b

c ≥ 2. Oleh karena itu

1a

+1b

+1c≥ 2 + 2 + 2 + 3 = 9.

Contoh 3 Buktikan bahwa jika a dan b bilangan real positif maka√a2 + b2

2≥ a + b

2≥√

ab ≥ 21a + 1

b

.

Selanjutnya, ketaksamaan ini akan berlaku sebagai kesamaan jika dan hanya jika a = b.

Bukti: Perhatikan bahwaa2 − 2ab + b2 = (a− b)2 ≥ 0. (1)

Hal ini ekivalen dengana2 + b2 ≥ 2ab. (2)

Ditambah dengan a2 + b2 untuk kedua ruas, Ketaksamaan (2) ekivalen dengan

2(a2 + b2

)≥ a2 + 2ab + b2 = (a + b)2,

5

Page 9: Materi-OSN-MatematikaSMA

yang ekivalen dengan √a2 + b2

2≥ a + b

2. (3)

Perhatikan bahwa Ketaksamaan (3) merupakan kesamaan jika dan hanya jika Ketaksamaan (1)merupakan kesamaan. Hal ini terjadi jika dan hanya jika a = b.

Dengan mengganti a dan b pada Ketaksamaan (2) berturut-turut dengan√

a dan√

b diperoleh

a + b ≥ 2√

ab,

yang ekivalen dengana + b

2≥√

ab. (4)

Dengan demikian, Ketaksamaan (4) merupakan kesamaan jika dan hanya jika√

a =√

b, atau a = b.Ketaksamaan (4) ekivalen dengan

2a + b

≤ 1√ab

,

yang ekivalen dengan2

1a + 1

b

=2

a + bab ≤ 1√

abab =

√ab. (5)

Selanjutnya ketaksamaan (5) merupakan kesamaan jika dan hanya jika a = b.Dari Ketaksamaan (3), (4), dan (5) diperoleh√

a2 + b2

2≥ a + b

2≥√

ab ≥ 21a + 1

b

.

Selanjutnya, ketaksamaan di atas akan merupakan kesamaan jika dan hanya jika a = b. �

Untuk bilangan real positif a dan b,√

a2+b2

2 , a+b2 ,√

ab, dan 21a+ 1

b

pada contoh di atas berturut-

turut disebut rataan kuadrat (QM), rataan aritmatika (AM), rataan geometri (GM), dan rataanharmonik (HM) dari a dan b. Dengan demikian, untuk bilangan real positif a dan b kita mempunyaiQM ≥ AM ≥ GM ≥ HM, selanjutnya QM = AM = GM = HM jika dan hanya jika a = b.

Rataan kuadrat, rataan aritmatika, rataan geometri, dan rataan harmonik dari n bilangan realpositif a1, a2, . . . , an berturut-turut adalah

QM =

√a2

1 + a22 + . . . + a2

n

n

AM =a1 + a2 + . . . + an

n,

GM = n√

a1a2 . . . an, dan

HM =n

1a1

+ 1a2

+ . . . + 1an

.

Teorema berikut merupakan perumuman dari Contoh 3.

Teorema 1 Jika QM, AM, GM, dan HM berturut-turut menyatakan rataan kuarat, rataan arit-matika, rataan geoetri, dan rataan harmonik dari bilangan real positif a1, a2, . . . , an, maka QM ≥AM ≥ GM ≥ HM . Selanjutnya, ketaksamaan ini akan berlaku sebagai kesamaan jika dan hanyajika a1 = a2 = . . . = an.

6

Page 10: Materi-OSN-MatematikaSMA

Contoh 4 Contoh 2 dikerjakan dengan menggunakan ketaksamaan AM-HM.

Menurut ketaksamaan AM-HM, kita mempunyai

31a + 1

b + 1c

≤ a + b + c

3=

13.

Karena 1a + 1

b + 1c > 0 maka 1

a + 1b + 1

c ≥ 9.

Contoh 5 Misalkan a, b, dan c adalah bilangan real positif yang memenuhi (1+a)(1+b)(1+c) = 8.Buktikan bahwa abc ≤ 1. Selanjutnya tentukan kapan kesamaan terjadi.

Penyelesaian: Dari yang diketahui diperoleh

8 = (1 + a)(1 + b)(1 + c) = 1 + (a + b + c) + (ab + bc + ca) + abc. (6)

Menurut ketaksamaan AM-GM,

a + b + c ≥ 3(abc)13 dan ab + bc + ca ≥ 3(abc)

23 , (7)

masing-masing ketaksamaan di atas merupakan kesamaan jika dan hanya jika a = b = c. Dari (6)dan (7) diperoleh

8 ≥ 1 + 3(abc)13 + 3(abc)

23 + abc =

(1 + (abc)

13

)3,

yang ekivalen dengan1 + (abc)

13 ≤ 3

√8 = 2,

atau(abc)

13 ≤ 2− 1 = 1,

yaituabc ≤ 1,

dan kesamaan terjadi jika dan hanya jika a = b = c = 1.

3 Sukubanyak (Polinom)

Misalkan F menyatakan sistem bilangan real atau sistem bilangan rasional dan n adalah bilanganbulat tidak negatif. Bentuk

f(x) = a0 + a1x + a2x2 + · · ·+ anxn

dengan a0, a1, a2, · · · , an di F dan an 6= 0, disebut sukubanyak atas F berderajad n. Himpunansemua sukubanyak atas F ditandai dengan F [x]. Berikut diberikan beberapa sifat sukubanyakyang sering digunakan.

Teorema 2 (Algoritma Pembagian) Misalkan f(x) dan g(x) di F [x] dan g(x) bukan sukubanyaknol. Maka terdapat sukubanyak q(x) dan r(x) di F [x] yang tunggal dan memenuhi

f(x) = q(x)g(x) + r(x)

dengan r(x) merupakan sukubanyak nol atau r(x) bukan sukubanyak nol yang berderajad kurangdari derajad g(x).

7

Page 11: Materi-OSN-MatematikaSMA

Dalam teorema di atas, q(x) disebut hasilbagi dan r(x) disebut sisa pembagian. Selanjutnyajika r(x) merupakan sukubanyak nol maka dikatakan f(x) habis dibagi oleh g(x).

Teorema 3 (Teorema Sisa) Jika sukubanyak f(x) dibagi oleh (x−a) maka sisanya adalah f(a).

Bilangan a di F disebut akar dari sukubanyak f(x) jika f(a) = 0. Sebagai akibat dari teoremadi atas diperoleh teorema berikut.

Teorema 4 (Teorema Faktor) Sukubanyak f(x) habis dibagi oleh (x− a) jika dan hanya jika amerupakan akar dari f(x).

Contoh 6 Sukubanyak f(x) = 2x3 +Ax2 +x−B habis dibagi oleh (x−2) dan bersisa -9 jika dibagi(x + 1). Tentukan nilai A dan B.

Penyelesaian: Karena f(x) habis dibagi oleh (x− 2) maka

0 = f(2) = 2(2)3 + A(2)2 + 2−B = 4A−B + 18. (8)

Karena f(x) bersisa -9 jika dibagi (x + 1) maka

−9 = f(−1) = 2(−1)3 + A(−1)2 + (−1)−B = A−B − 3. (9)

Dengan menyelesaikan sistem persamaan di atas (mengeliminasi (8) dan (9) serta mensubstitusikanke salah satu darinya) diperoleh A = −4 dan B = 2.

Contoh 7 Tentukan semua bilangan asli n sehingga sukubanyak x2 + x + 1 membagi sukubanyakx2n + xn + 1.

Penyelesaian:Perhatikan bahwa x3 − 1 = (x− 1)(x2 + x + 1) dan x3 − 1 membagi x3m − 1.

(i) Untuk n = 3k,x2n + xn + 1 = x6k + x3k + 1

= (x6k − 1) + (x3k − 1) + 3= (x2 + x + 1)Q(x) + 3.

(ii) Untuk n = 3k + 1,x2n + xn + 1 = x6k+2 + x3k+1 + 1

= x2(x6k − 1) + x(x3k − 1) + x2 + x + 1= (x2 + x + 1)R(x).

(iii) Untuk n = 3k + 2,x2n + xn + 1 = x6k+4 + x3k+2 + 1

= x4(x6k − 1) + x2(x3k − 1) + x4 + x2 + 1= x4(x6k − 1) + x2(x3k − 1) + x(x3 − 1) + x2 + x + 1= (x2 + x + 1)S(x).

8

Page 12: Materi-OSN-MatematikaSMA

Jadi x2 + x + 1 membagi x2n + xn + 1 jika dan hanya jika n bukan kelipatan 3.

Sifat yang lain dari sukubanyak yang sering digunakan adalah sifat simetri akar, yang lebihdikenal dengan nama Teorema Vieta, yaitu hasil tambah dan hasil tambah dari hasil kali akar-akarsuatu sukubanyak.

(a) Jika sukubanyak ax2 + bx + c mempunyai akar-akar x1 dan x2 maka

ax2 + bx + c = a(x− x1)(x− x2) = ax2 − a(x1 + x2)x + ax1x2.

Sehingga

x1 + x2 = − b

adan x1x2 =

c

a.

(b) Misalkan x1, x2, dan x3 akar-akar sukubanyak ax3 + bx2 + cx + d. Dengan ekspansi

a(x− x1)(x− x2)(x− x3) = ax3 − a(x1 + x2 + x3)x2 + a(x1x2 + x2x3 + x3x1)x− ax1x2x3

dan komparasi koefisien diperoleh

x1 + x2 + x3 = − b

a, x1x2 + x2x3 + x3x1 =

c

a, dan x1x2x3 = −d

a.

Contoh 8 Misalkan x1, x2, dan x3 akar-akar dari x3 + 3x2 − 7x + 1. Tentukan x21 + x2

2 + x23.

Penyelesaian: Menurut Teorema Vieta,

x1 + x2 + x3 = −3 dan x1x2 + x2x3 + x3x1 = −7.

Sehingga

9 = (x1 + x2 + x3)2

= x21 + x2

2 + x23 + 2 (x1x2 + x2x3 + x3x1)

= x21 + x2

2 + x23 + 2(−7).

Oleh karena itu x21 + x2

2 + x23 = 23.

4 Sistem Persamaan

Bentuk yang melibatkan variabel, yaitu

f(x1,x2, . . . , xn) = c

disebut persamaan dengan n buah variabel. Sistem persamaan adalah suatu sistem yang terdiridari dua atau lebih persamaan, yaitu

f1(x1,x2, . . . , xn) = c1

f2(x1,x2, . . . , xn) = c2

...fm(x1,x2, . . . , xn) = cm

9

Page 13: Materi-OSN-MatematikaSMA

Sistem persamaan di atas disebut sistem persamaan dengan n buah variabel dan m persamaan.Solusi dari suatu sistem persamaan adalah solusi secara simultan dari semua persamaan di dalamsistem itu. Cara baku untuk mencari solusi suatu sistem persamaan dengan cara eliminasi danatau substitusi. Berikut akan diberikan beberapa contoh soal yang tidak regular.

Contoh 9 Cari semua solusi real dari sistem persamaan

x +2x

= 2y

y +2y

= 2z

z +2z

= 2x.

Penyelesaian: Misalkan (x, y, z) solusi sistem persamaan di atas. Diantara x, y, dan z tidakmungkin ada yang nol. Perhatikan bahwa jika salah satu positif maka dua yang lain juga positif.Selanjutnya, dengan mengalikan dengan -1 akan diperoleh solusi yang lain. Asumsikan x, y, z > 0.Dengan menggunakan ketaksamaan AM-GM untuk masing-masing persamaan diperoleh

2y = x +2x≥ 2

√x

(2x

)= 2√

2⇐⇒ y ≥√

2,

2z = y +2y≥ 2

√y

(2y

)= 2√

2⇐⇒ z ≥√

2,

2x = z +2z≥ 2

√z

(2z

)= 2√

2⇐⇒ x ≥√

2.

Dengan menambahkan semua persamaan dari sistem persamaan semula dan hasil di atas, diperoleh

3√

2 ≤ x + y + z = 2(

2x

+2y

+2z

)≤ 3√

2.

Dengan demikan haruslah x = y = z =√

2. Selanjutnya dapat ditunjukkan bahwa(√

2,√

2,√

2)

dan(−√

2,−√

2,−√

2)

adalah solusi yang dimaksud.

Contoh 10 Cari semua solusi real dari sistem persamaan

4x2

4x2 + 1= y

4y2

4y2 + 1= z

4z2

4z2 + 1= x.

Penyelesaian: Perhatikan fungsi f : [0,∞) −→ [0,∞),

f(t) =4t2

4t2 + 1,

merupakan fungsi monoton naik murni.Oleh karena itu jika x < y maka y = f(x) < f(y) = z.Akibatnya z = f(y) < f(z) = x. Sehingga x < y < z < x, suatu yang tidak mungkin. Dengan cara

10

Page 14: Materi-OSN-MatematikaSMA

yang sama jika x > y maka akan diperoleh suatu kontradiksi. Jadi x = y. Dengan menggunakanargumen yang sama diperoleh y = z. Jadi x = y = z. Dengan menyelesaikan persamaan

4t2

4t2 + 1= t

diperoleh t = 0 atau t = 12 . Jadi solusi dari sistem persamaan di atas hanyalah tripel (0, 0, 0) dan(

12 , 1

2 , 12

).

Contoh 11 Tentukan semua solusi real dari sistem persamaan{x2 + y2 + z2 = 1

x2y2 + y2z2 + z2x2 = 9x2y2z2

Penyelesaian: Dari persamaan pertama, tidak mungkin x = y = z = 0. Dari persamaankedua tidak mungkin jika satu variabel nol dan dua variabel tidak nol.

Kasus I: Jika dua variabel nol dan satu variabel tidak nol. Misalkan x = y = 0, z 6= 0. Diperolehz = ±1. Dengan demikian (0, 0,±1) merupakan solusi. Dengan cara yang sama diperoleh (0,±1, 0)dan (±1, 0, 0) juga merupakan solusi.

Kasus II: Jika ketiga variabel tidak nol. Persamaan kedua ekivalen dengan

1x2

+1y2

+1z2

= 9.

Digunakan AM-HM, persamaan pertama, dan persamaan di atas diperoleh

13

=x2 + y2 + z2

3≥ 3

1x2 + 1

y2 + 1z2

=39

=13.

Jadi x2+y2+z2

3 = 31

x2 + 1y2 + 1

z2. Oleh karena itu x2 = y2 = z2. Diperoleh (±1

3

√3,±1

3

√3,±1

3

√3) solusi

sistem persamaan di atas.

Soal-soal Latihan

1. Diketahui a + b = 1 dan a2 + b2 = 2. Tentukan a4 + b4.

2. Sederhanakan√

9 +√

80−√

9−√

80 (tanpa melibatkan tanda akar).

3. Buktikan bahwa jika a, b, c ∈ R, dan a2 + b2 + c2 = 1 maka −12 ≤ ab + bc + ca ≤ 1.

4. Tentukan bilangan real a agar hasil tambah kuadrat akar-akar x2 +(a−2)x−a−3 minimum.

5. Hitung∑n

k=1 k!(k2 + k + 1).

6. Tentukan jumlah dari√1 +

112

+122

+

√1 +

122

+132

+ · · ·+√

1 +1

20012+

120022

7. Diketahui f(x) = 2x + 1 dan g(f(x)) = x2 + 3x + 1. Tentukan g(3).

11

Page 15: Materi-OSN-MatematikaSMA

8. Misalkan x dan y bilangan real dan x2 + 3xy + y2 = 60. Tentukan nilai maksimum yangmungkin untuk xy.

9. Misalkan semua akar dari x6 − 6x5 + ax4 + bx3 + cx2 + dx + 1 = 0 adalah positif. Tentukana, b, c, d.

10. Untuk bilangan real a, b, dan c yang memenuhi a ≥ b ≥ c > 0, buktikan bahwa

a2 − b2

c+

c2 − b2

a+

a2 − c2

b≥ 3a− 4b + c.

11. Buktikan bahwa115

<12.34.56.78· · · 99

100<

110

.

12. Misalkan a1, a2, . . . , an bilangan real positif dan b1, b2, . . . , bn adalah penataan kembali daria1, a2, . . . , an. Buktikan bahwa

a1

b1+

a2

b2+ · · ·+ an

bn≥ n.

13. Untuk bilangan asli n sebarang, buktikan bahwa

(a)(1 + 1

n

)n<

(1 + 1

n+1

)n+1.

(b)(1 + 1

n

)n+1>

(1 + 1

n+1

)n+2.

14. Buktikan bahwa

1√1 +

√3

+1√

5 +√

7+ · · ·+ 1√

9997 +√

9999> 24.

15. Buktikan bahwa n√

n + n√

n + n√

n− n√

n < 2 n√

n untuk n ≥ 2.

16. Misalkan x1 + x2 + x3 = π2 , xi > 0. Buktikan bahwa

sinx1 sin x2 sinx3 ≤18.

17. Buktikan bahwa tan a + tan b ≥ 2 tan√

ab untuk setiap a, b ∈ [0, π2 ).

18. Buktikan bahwa untuk bilangan asli n berlaku

n! ≤(

n + 12

)n

.

19. Misalkan a1, a2, . . . , an dan b1, b2, . . . , bn adalah bilangan real positif yang memenuhi a1a2 . . . an =b1b2 . . . bn. Buktikan bahwa

(a1b1 + 1) (a2b2 + 1) . . . (anbn + 1)b1b2 . . . bn

≥ 2n.

12

Page 16: Materi-OSN-MatematikaSMA

20. Cari semua solusi real dari sistem persamaan berikut.

x3 + y = 3x + 42y3 + z = 6y + 63z3 + x = 9z + 8

21. Cari semua tripel (x, y, z) yang memenuhi x4 + y4 + z4 − 4xyz = −1.

22. Cari semua solusi real dari sistem persamaan

x + y =√

4z − 1y + z =

√4x− 1

z + x =√

4y − 1

23. Buktikan tidak ada belangan real x, y, x yang memenuhi

x2 + 4yz + 2z = 0x + 2xy + 2z2 = 0

2xz + y2 + y + 1 = 0

24. Cari semua bilangan real m sehingga persamaan

(x2 − 2mx− 4(m2 + 1))(x2 − 4x− 2m(m2 + 1)) = 0

mempunyai tepat tiga akar.

Rujukan

[1] Engel, A. 1998. Problem-Solving Strategies. New York: Springer-Verlag.

[2] Larson, L. C. 1983. Problem-Solving Through Problems. New York: Springer-Verlag.

13

Page 17: Materi-OSN-MatematikaSMA

TEORI BILANGAN

Nanang SusyantoJurusan Matematika FMIPA UGM

1 Sistem Bilangan Bulat

1.1 Latar belakang

Pada zaman dahulu, manusia hanya mengenal sistem bilangan asli

N = {1, 2, 3, ...}

Mereka hanya mengenal sistem bilangan tersebut karena pada waktu itu yang mereka butuhkanhanyalah menghitung sesuatu yang mereka miliki atau mereka dapatkan. Sebagai contoh: menghi-tung hasil binatang buruan, menghitung banyak persediaan makanan yangmereka miliki, dan se-bagainya. Akan tetapi, setelah selang waktu tertentu mereka merasakan binatang buruannya habisyang kemudian dilambangkan dengan simbol ”0”. Oleh karena itu, mereka mulai mengenal sistembilangan cacah

N0 = {0} ∪ N = {0, 1, 2, 3, ...}

Seiring dengan adanya sistem barter, mereka menemui masalah ”jika setiap seekor kambing dapatditukar dengan 10 ekor ayam, maka bagaimana jika saya mempunyai 7 ekor ayam yang inginditukarkan dengan seekor kambing?”. Tentu saja orang tersebut masih hutang/kurang 3 ekorayam bukan? Kekurangan 3 ekor ini yang kemudian dilambangkan −3. Dari sini mereka mulaimengenal bilangan bulat

Z = {..,−3,−2,−1, 0, 1, 2, 3, ...}

Perkembangan selanjutnya, jika seseorang mempunyai 1 buah apel sedangkan ia punya dua anak,maka untuk menuliskan kejadian ini mereka memberi simbol 1

2 . Dengan demikian, mereka mulaimengenal sistem bilangan rasional

Q ={a

b|a dan b bilangan bulat dengan b 6= 0

}1.2 Mengingat kembali notasi pada himpunan, relasi serta operasi dua him-

punan

Secara umum, suatu himpunan kita notasikan dengan huruf kapital A,B, C, ... Dalam teori bi-langan kita hanya akan bekerja pada sistem bilangan asli, cacah, bulat, dan yang paling luas kitaakan bekerja pada sistem bilangan rasional. Untuk himpunan-himpunan tersebut, yaitu himpunanbilangan asli, cacah, bulat, dan rasional berturut-turut kita notasikan dengan N, N0, Z, dan Q.Sedangkan untuk anggota-anggota dari suatu himpunan biasanya kita tulis dengan huruf kecila, b, c, ... dan kita akan menggunakan lambang ∈ untuk menyatakan anggota/elemen dan /∈ untukbukan anggota. sebagai contoh: Misalkan A = {1, 2, 3}, maka 1 ∈ A tetapi 5 /∈ A.

14

Page 18: Materi-OSN-MatematikaSMA

1.2.1 Relasi dua himpunan

1. Himpunan A dikatakan himpunan bagian dari B kita tulis dengan A ⊆ B jika untuk setiapx ∈ A maka x ∈ B.

2. Himpunan A dikatakan sama dengan himpunan B kita tulis dengan A = B jika A ⊆ B danB ⊆ A.

3. Himpunan B dikatakan komplemen dari himpunan A kita tulis dengan B = Ac atau B = A′

jika himpunan B berisi semua anggota dari himpunan semesta yang bukan anggota himpunanA.

1.2.2 Operasi dua himpunan

1. Irisan dua himpunan A dan B dinotasikan dengan A ∩ B adalah himpunan yang anggota-angotanya merupakan anggota dari kedua himpunan A dan B. Secara matematika dapatkita tuliskan

A ∩B = {x|x ∈ A dan x ∈ B}

2. Gabungan dua himpunan A dan B dinotasikan dengan A∪B adalah himpunan yang anggota-angotanya merupakan anggota himpunan A atau himpunan B. Secara matematika dapat kitatuliskan

A ∪B = {x|x ∈ A atau x ∈ B}

3. Selisih dua himpunan A dan B dinotasikan dengan A − B adalah himpunan yang anggota-anggotanya merupakan anggota himpunan A tetapi bukan anggota himpunan B. Secarametematika dapat kita tuliskan

A−B = {x|x ∈ A dan x /∈ B}

Definisi 1 (sifat tertutup)

Himpunan A dikatakan tertutup terhadap operasi ∗ (bisa penjumlahan, pengurangan, pemba-gian, perkalian, dan lain-lain) jika untuk setiap a, b ∈ A berlaku a ∗ b ∈ A.

Contoh 1.Himpunan bilangan bulat tertutup terhadap penjumlahan, pengurangan, dan perkalian, tetapi

tidak terhadap pembagian karena 12 /∈ Z.

1.3 Himpunan bilangan bulat

Tentu saja himpunan ini telah kita kenal dengan akrab sejak di sekolah dasar. Di sini kita akanmembahas sifat-sifat yang berkaitan dengan himpunan bilangan bulat dan himpunan bagiannya.Salah satu himpunan bagian dari himpunan bulat adalah himpunan bilangan asli, himpunan iniberanggotakan bilangan-bilangan bulat yang positif.

Beberapa sifat yang berkaitan dengan bilangan bulat dan himpunan bagiannya

1. Himpunan bilangan bulat tertutup terhadap penjumlahan, pengurangan, dan perkalian,

2. Himpunan bilangan asli tertutup tertutup terhdap penjumlahan dan perkalian,

15

Page 19: Materi-OSN-MatematikaSMA

3. Setiap himpunan bagian dari himpunan bilangan asli selalu mempunyai elemen terkecil (min-imal),

4. Setiap himpunan bagian dari himpunan bilangan bulat yang berhingga selalu mempunyaielemen minimal dan elemen maksimal,

Sifat (3) disebut sifat terurut rapi (well ordering principle).

1.4 Soal-soal Latihan

1. Tunjukkan hukum D’Morgan yaitu (A ∩B)c = Ac ∪Bc dan (A ∪B)c = Ac ∩Bc.

2. Misalkan A ⊆ Z tertutup terhadap pengurangan. Jika diketahui 4 dan 7 merupakan anggotaA,

(a) tunjukkan bahwa 0, 100, 208 ∈ A,

(b) daftarlah semua anggota dari A.

3. Misalkan S adalah himpunan yang memuat semua bilangan bulat. Jika untuk setiap s ∈ Syang tidak nol, terdapat s′ ∈ S sehingga ss′ = 1, maka tentukan semua anggota-anggota S.

4. Jumlah dari rata-rata aritmatik himpunan A dan rata-rata aritmatik himpunan B adalah5002. Himpunan A dan B terdiri dari bilangan-bilangan asli berurutan. Jika A∩B = {2005}tentukan kemungkinan unsur himpunan B yang terbesar. (soal olimpiade matematika tkpropinsi tahun 2005)

5. Misalkan S adalah himpunan yang memuat bilangan 1, 2, 3, dan 4. Diketahui untuk sebaranga, b, c, d ∈ S yang semuanya berbeda akan berlaku ab + cd ∈ S. Selidiki apakah 2008 ∈ S?

6. Buktikan sifat well ordering principle pada sebarang sub himpunan bilangan asli.

7. Suatu barisan bilangan bulat {an} memenuhi persamaan aan+n = an untuk setiap bilanganasli n. Jika diketahui a2008 = 1, maka tunjukkan bahwa an = 1 untuk setiap bilangan asli n.

2 Teorema Keterbagian

Pada bab ini, kita akan mempelajari tentang konsep dasar keterbagian, algoritma pembagian,faktor persekutuan terbesar, dan kelipatan persekutuan terkecil.

2.1 Keterbagian

Definisi 1Misalkan a dan b adalah bilangan bulat dengan a 6= 0. Bilangan a dikatakan habis membagi b

jika terdapat bilangan bulat k sehingga b = ka. Untuk selanjutnya kita tulis a|b, sedangkan dalamhal a tidak habis membagi b kita tulis dengan a - b.

Contoh 13|12 karena terdapat bilangan bulat k yaitu k = 4 sehingga 12 = 4× 33 - 7 karena kita tidak mungkin mendapatkan bilangan bulat k sehinga 7 = k × 3.Dari definisi di atas kita dapat menurunkan sefat-sifat sebagai berikut

16

Page 20: Materi-OSN-MatematikaSMA

Sifat 1Untuk setiap bilangan bulat a yang tidak nol selalu berlaku a|a dan a|0Sifat 2Untuk setiap bilangan bulat a selalu berlaku 1|aSifat 3Jika a|b maka

(i) |a| ≤ |b|

(ii) ac|bc untuk setiap bilangan bulat c yang tidak nol.

Sifat 4Jika a|b dan a|c maka a| (mb + nc) untuk setiap bilangan bulat m dan n.

Di sini, kita hanya akan membuktikan Sifat 4. Sedangkan untuk sifat-sifat yang lainnya diser-ahkan kepada pembaca sebagai latihan.

Bukti Sifat 4Perhatikan bahwa a|b artinya terdapat bilangan bulat k sehingga b = ka, dan juga kita tahu

a|c yang berarti terdapat bilangan bulat l sehingga c = la. Dari kedua fakta tersebut kita punyamb + nc = mka + nla = (mk + nl) a yang berarti a| (mb + nc) �

Contoh 2Tentukan semua bilangan asli n sehingga 3n+25

2n−5 juga merupakan bilangan asli (soal OSN tkPropinsi, 2002)

Jawab:Agar 3n+25

2n−5 merupakan bilangan bulat haruslah 2n − 5|3n + 25, di lain pihak kita juga punya2n− 5|2n− 5. Dengan demikian 2n− 5| (2 (3n + 25)− 3 (2n− 5)) atau ekivalen dengan 2n− 5|65.Dari sini kita simpulkan 2n−5 = 1, 5, 13, atau 65. Yang selanjutnya kita dapatkan solusi n = 3, 5, 9,atau 35.

2.2 Algoritma Pembagian

Teorema 1Jika a dan b adalah bilangan bulat dan b > 0, maka terdapat dengan tunggal bilangan bulat q

dan r sehinggaa = bq + r

dengan 0 ≤ r < b.

Bukti:Padang himpunan ..., a−3b, a−2b, a− b, a, a+ b, a+2b, a+3b, ... Jika barisan tersebut memuat

unsur nol, maka terdapat bilangan bulat q sehingga a = bq + r dengan r = 0. Jika barisan tersebuttidak terdapat unsur nol, maka a tidak mungkin nol. Jika a > 0 maka a + ab = a (1 + b) > 0, danjika a < 0 maka a − ab = −a (b− 1) > 0. Jadi, barisan tersebut memuat unsur positif. Dengandemikian, jika kita himpun semua elemen yang positif sebut saja himpunan S, maka menurut wellordering principle S mempunyai elemen terkecil, sebut elemen minmal tersebut adalah r = a− qb.Kita akan buktikan bahwa r < b. Jelas r 6= b (mengapa?), andaikan r > b maka akan kita peroleh

17

Page 21: Materi-OSN-MatematikaSMA

s = a−(q + 1) b = a−qb−b = r−b > 0. Perhatikan bahwa s ∈ S, dan s < r. Ini kontradiksi denganasumsi bahwa r merupakan elemen terkecil. Jadi haruslah terdapat bilangan bulat q sehingga

0 < r = a− bq < b

atau dengan kata laina = bq + r dengan 0 < r < b

Sekarang akan kita buktikan ketunggalannya. Misalkan terdapat bilangan bulat 0 ≤ r1, r2 < b danq1 serta q2 sehingga a = bq1 + r1 = bq2 + r2. Dari sini akan diperoleh b (q1 − q2) = r2 − r1 yangberarti b| (r2 − r1) , akan tetapi −b < r2 − r1 < b , akibatnya r2 − r1 = 0 atau dengan kata lainr2 = r1. Dengan fakta r2 = r2 ini juga akan berakibat q1 = q2 dan kita selesai.�

Dari teorema di atas, dapat kita pahami bahwa jika m suatu bilangan asli, maka untuk sebarangbilangan bulat n dapat dinyatakan sebagai

n = mk + r

untuk suatu bilangan bulat k dan r dengan 0 ≤ r ≤ m − 1. Bilangan yang berbentuk mk + radalah bilangan bulat yang bersisa r ketika dibagi m. Sebagai contoh, kita kita ambil m = 2,maka fakta di atas mengatakan bahwa setiap bilangan bulat dapat dinyatkan dalam bentuk 2katau 2k + 1, yang selanjutnya dalam kehidupan kita sehari-hari bilangan yang berbentuk 2k dan2k+1 berturut-turut kita katakan bilangan genap dan bilangan ganjil. Sekarang, marilah kita lihatbeberapa contoh berikut:

Contoh 3Tentukan semua banyak bilangan asli n dengan n < 2008 yang menyebabkan 1

3n (n + 1) meru-pakan bilangan bulat.

Jawab:Setiap bilangan asli dapat dinyatakan dalam bentuk 3k, 3k + 1, atau 3k + 2Untuk n = 3k, kita punya 1

3 .3k. (3k + 1) = k (3k + 1) merupakan bilangan bulat,untuk n = 3k + 1 kita punya 1

3 (3k + 1) (3k + 2) = 3k2 + 3k + 23 bukan merupakan bilangan

bulat,untuk n = 3k + 2 kita punya 1

3 (3k + 2) (3k + 3) = (3k + 2) (k + 1) merupakan bilangan bulat.Jadi, bilangan asli n yang menyebabkan 1

3n (n + 1) bukan bilangan bulat adalah bilangan asliyang berbentuk 3k + 1. Bilangan seperti ini yang kurang dari 2008 dapat kita daftar dengan caraberikut:

1 = 3.0 + 1, 4 = 3.1 + 1, 7, ..., 2005 = 3.668 + 1

yang berarti ada 669 bilangan asli kurang dari 2008 yang berbentuk 3k + 1. Dengan ddemikian,banyak bilangan asli n < 2008 yang menyebabkan 1

3n (n + 1) merupakan bilangan bulat adalah2007− 669 = 1338.

Contoh 4.Tunjukkan bahwa tidak ada bilangan kuadrat pada barisan

11, 111, 1111, 11111, ...

Jawab:Perhatikan bahwa untuk n = 2k kita punya n2 = 4k2 dan untuk n = 2k + 1 kita punya

n2 = 4(k2 + k

)+ 1. Dari sini kita dapat simpulkan bahwa sisa pembagian dari bilangan kadrat

oleh 4 adalah 0 atau 1. Sekarang perhatikan barisan

11, 111, 1111, 11111, ...

18

Page 22: Materi-OSN-MatematikaSMA

sisa pembagian setiap suku oleh 4 selalu bersisa 3, dengan demikian tidak ada bilangan kuadratpada barisan di atas.

2.3 Pembagi sekutu terbesar dan faktor sekutu terkecil

2.3.1 Pembagi sekutu terbesar

Pada saat sekolah dasar, kita semua tentu telah mengenal pembagi sekutu terbesar atau biasadisebut faktor persekutuan terbesar (FPB), atau disebut juga greatest common divisor (gcd).

Definisi 2.Diberikan a dan b adalah bilangan bulat yang tidak keduanya nol. Bilangan asli d disebut

pembagi sekutu terbesar dari a dan b atau ditulis dengan d = gcd (a, b) jika

(i). d|a dan d|b

(ii). untuk setiap bilangan asli c dengan c|a dan c|b haruslah berlaku c ≤ d.

bagian (i) mengatakan bahwa d adalah pembagi sekutu dari a dan b, sedangkan bagian (ii)mengatakan bahwa untuk setiap pembagi sekutu dari a dan b harus lebih kecil atau sama dengand, dengan kata lain (ii) mengatakan bahwa d merupakan pembagi sekutu yang terbesar.

Definisi 3.Bilangan bulat a dan b dikatakan saling prima (relatif prima) jika gcd (a, b) = 1.Definisi 4.Untuk sebarang bilangan bulat a, b dan c didefinisikan

gcd (a, b, c) = gcd (gcd (a, b) , c) = gcd (a, gcd (b, c))

Dari definisi di atas, dapat diturunkan beberapa sifat di bawah ini:

1. gcd (a, b) = gcd (b, a) = gcd (|a|, |b|)

2. gcd (a, 1) = 1 untuk setiap bilangan bulat a,

3. gcd (a, 0) = |a| untuk setiap bilangan bulat tak nol a,

4. gcd (ma,mb) = |m| gcd (a, b) untuk setiap bilangan bulat tak nol m,

5. jika d = gcd (a, b) maka gcd(

ad , b

d

)= 1.

Bukti untuk sifat (5) dapat dilihat pada contoh 6, sedangkan untuk yang lain diserahkan kepadapembaca sebagi latihan.

19

Page 23: Materi-OSN-MatematikaSMA

2.3.2 Menentukan gcd dua bilangan dengan algoritma Euclide

Misalkan a dan b bilangan bulat yang tidak keduanya nol. Kita akan menghitung gcd dari a dan bdengan menggunakan algoritma pembagian yang telah kita kenal pada sub bab sebelumnya. Karenagcd (a, b) = gcd (b, a) = gcd (|a|, |b|) , maka di sini hanya akan dibahas untuk a dan b bilangan aslidengan a > b.

Berdasarkan algoritma pembagian, akan terdapat bilangan bulat q dan r dengan 0 ≤ r < bsehingga a = bq + r atau ekivalen dengan r = a− bq.

Perhatikan bahwa untuk setiap pembagi sekutu a dan b pasti merupakan pembagi dari r. Olehsebab itu, dapat kita simpulkan gcd (a, b) = gcd (b, r) . Jika r = 0, maka gcd (a, b) = gcd (b, 0) = b.Jika r 6= 0 kita dapat lakukan langkah yang sama pada b dan r, yakni terdapat q1 dan r1 dengan0 ≤ r1 < r sehingga b = rq1 + r1. Dengan argumen yang sama seperti sebelumnya, kita simpulkangcd (b, r) = gcd (r, r1) . Jika r1 = 0, maka gcd (b, r) = gcd (r, r1) = r. Jika tidak, kita dapatmelakukan langkah di atas sehingga kita peroleh barisan r1, r2, ... Akan tetapi, karena a dan bberhingga, maka tentu akan terdapat n sehingga rn = 0. Dengan demikian

gcd (a, b) = gcd (b, r) = gcd (r, r1) = gcd (r1, r2) = ... = gcd (rn−1, rn) = gcd (rn−1, 0) = rn−1

Untuk lebih jelasnya, perhatikan contoh berikut:Contoh 5.Hitung gcd (2008, 123456) .Jawab:gcd (2008, 123456) = gcd (123456, 2008) . Dengan algoritma pembagian 123456 = 61×2008+968,

dengan demikian gcd (123456, 2008) = gcd (2008, 968) . Kemudian 2008 = 2 × 968 + 72, sehinggagcd (2008, 968) = gcd (968, 72) , seterusnya dengan algoritma pembagian akan kita peroleh

gcd (968, 72) = gcd (72, 32) = gcd (32, 8) = gcd (8, 0) = 8

dengan demikian gcd (2008, 123456) = 8.Contoh 6.Jika d = gcd (a, b) maka tunjukkan bahwa gcd

(ad , b

d

)= 1.

Jawab:Misalkan gcd

(ad , b

d

)= k, maka kita punya k|ad dan k| bd , yang berakibat kd|a dan kd|b dan

selanjutnya kita peroleh kd ≤ d (mengapa?). Dengan demikian, k ≤ 1 dan kita peroleh k = 1(karena k merupakan bilangan asli). Jadi gcd

(ad , b

d

)= 1.

Teorema 2(Identitas Benzout)Jika d = gcd (a, b) maka terdapat bilangan bulat x dan y sehingga ax + by = d.Jawab:Bentuk himpunan

S = {ax + by|x, y ∈ Z, ax + by > 0}

perhatikan bahwa jika kita ambil x = a dan n = b kita punya a2 + b2 > 0 (ingat pada pendefinisiangcd kita asumsikan a dan b tidak keduanya nol) yang berarti S tidak kosong. Dengan demikian Smempunyai elemen terkecil, sebut saja d. Kita akan buktikan bahwa d = gcd (a, b) . Pertama akankita buktikan bahwa d|a dan d|b. Dengan algoritma pembagian kita dapat tulis a = dq + r dengan0 ≤ r < t atau dengan kata lain r = a − dq. Akan tetapi d ∈ S yang berarti d = am + bn untuksuatu m, n ∈ Z. Oleh karena itu, kita punya r = a − dq = a − (am + bn) q = a − aqm − bnq =a (1− qm)− b (nq) ∈ S. Karena d adalah elemen terkecil dari S dan r < t maka r = 0 yang berarti

20

Page 24: Materi-OSN-MatematikaSMA

d|a, dengan cara yang sama kita peroleh juga d|b. Sekarang misalkan c adalah sebarang bilanganasli dengan c|a dan c|b, maka c|am + bn atau c|d. terbukti bahwa d = gcd (a, b) .

Dengan menggunakan teorema di atas kita dapat menurunkan beberapa sifat sebagai berikut:

1. Jika d = gcd (a, b) maka untuk sebarang bilangan bulat c dengan c|a dan c|b haruslah berlakuc|d.

2. Jika a|bc dan gcd (a, b) = 1, maka a|c.

Bukti:Untuk sifat (1) dapat langsung dilihat dari pembuktian teorema identitas Benzout, sehingga di

sini hanya akan kita buktikan untuk sifat (2) .Bukti sifat (2) :Perhatikan bahwa a|bc, artinya terdapat bilangan bulat k sehingga bc = ka. Selain itu, kita juga

punya gcd (a, b) = 1. Menurut identitas Benzout kita dapat menemukan bilangan bulat x dan ydengan sifat ax + by = 1. Dengan mengalikan kedua ruas dengan c akan kita peroleh acx + bcy = cyang ekivalen dengan acx = c− bcy. Perhatikan bahwa a| (c− bcy) , akan tetapi karena a|bcy akanberakibat a|c.�

Contoh 7.Diberikan gcd (15, 24) = 3. Cari salah satu pasangan bulat (x, y) sehingga 15x + 24y = 3.Jawab:Perhatikan bahwa 24 = 15.1 + 9, 15 = 9.1 + 6, dan 9 = 6.1 + 3 (ingat mencari gcd dengan

Algoritma Euclide). Dengan demikian

3 = 9− 6 = 9− (15− 9) = 2.9− 15 = 2 (24− 15)− 15 = −3.15 + 2.24

kita dapat mengambil x = −3 dan y = 2.Contoh 8Jika gcd (a, b) = 1 dan gcd (a, c) = 1, tunjukkan bahwa gcd (a, bc) = 1.Jawab:kita punya gcd (a, b) = 1 dan gcd (a, c) = 1, sehingga ada bilangan bulat x dan y yang memenuhi

ax + by = 1 dan ada bilangan bulat m dan n yang memenuhi ax + cy = 1. Kemudian kita peroleh(ax + by) (am + cn) = 1 ⇐⇒ a (axm + bmy + cnx) + bc (ny) = 1 yang berakibat gcd (a, bc) |1 dantentunya gcd (a, bc) = 1.

Kelipatan persekutuan terkecil. Selain pembagi sekutu terbesar, tentunya pada saat sekolahdasar juga kita telah mengenal kelipatan persekutuan terkecil (KPK). Dalam pembahasan selan-jutnya, untuk sebarang bilangan bulat a dan b KPK dari a dan b kita tulis dengan [a, b] .

Definisi 5Diberikan bilangan bulat a dan b yang tidak keduanya nol. Bilangan bulat positif m disebut

KPK dari a dan b jika

1. m|a dan m|b,

2. untuk setiap bilangan bulat positif n dengan n|a dan n|b haruslah berlaku m ≤ n.

21

Page 25: Materi-OSN-MatematikaSMA

Definisi 6Misalkan a, b, dan c bilangan bulat yang tidak semuanya nol, KPK dari a, b, dan c didefinisikan

sebagai[a, b, c] = [[a, b] , c] = [a, [b, c]]

Langsung dari definisi di atas, kita dapat menurunkan beberapa sifat sederhana sebagai berikut:

1. [a, b] = [b, a] untuk setiap bilangan bulat a dan b yang tidak keduanya nol,

2. [a, 0] = 0 untuk setiap bilangan bulat tak nol a,

3. [a, 1] = |a| untuk setiap bilangan bulat a.

Teorema 3Jika a dan b adalah bilangan bulat yang tidak keduanya nol, maka

[a, b] =ab

gcd (a, b)

Bukti:Misalkan d = gcd (a, b) , maka a = da1 dan b = db1 untuk suatu bilangan bulat a1, b1 dengan

gcd (a1, b1) = 1. Misalkan m = da1b1. Akan kita buktikan bahwa m = [a, b] . Jelas bahwa a|m danb|m. Ambil sebarang bilangan asli n dengan sifat a|n dan b|n, artinya n = ka dan n = lb untuksuatu bilangan bulat k, l. Dari sini kita dapatkan ka = lb ⇐⇒ kda1 = ldb1 ⇐⇒ ka1 = lb1. Kitapunya a1|lb1, dan karena gcd (a1, b1) = 1 maka a1|l yang berarti l = ta1 untuk suatu bilanganbulat t. Dengan demikian kita punya n = lb = ldb1 = tda1b1, akibatnya m|n dan kita selesaimembuktikan m = [a, b] .�

Contoh 9.Hitung [56, 72] .Jawab:Karena gcd (56, 72) = 8 maka [56, 72] = 56.72

8 = 504.

Contoh 10.Tentukan bilangan bulat positif terkecil lebih dari 1 yang bersisa 1 ketika dibagi k untuk setiap

2 ≤ k ≤ 10.Jawab:Bilangan yang bersisa 1 ketika dibagi k pasti berbentuk km + 1 untuk suatu bilangan bulat m,

dan karena bilangan tersebut harus berbentuk km + 1 untuk setiap 2 ≤ k ≤ 10 maka bilangantersebut harus berbentuk rm + 1 dengan r habis dibagi 2 ≤ k ≤ 10. Dengan demikian, r =[2, 3, 4, ..., 10] = 2520. Jadi bilangan yang dimaksud pasti berbentuk 2520m+1 untuk suatu bilanganbulat m. Dan karena kita mencari yang terkecil dan lebih besar dari 1 maka kita ambil m = 1.Jadi, bilangan yang dimaksud adalah 2521.

2.4 Soal-soal Latihan

1. Tentukan semua bilangan bulat p yang menyebabkan

(a) 8p+92p+1 merupakan bilangan bulat,

22

Page 26: Materi-OSN-MatematikaSMA

(b) 2p + 1 membagi 2p2 + 7,

(c) p2 − 10 kelipatan p + 10.

2. Tentukan semua bilangan asli n sehingga n3+24n+3 juga merupakan bilangan asli.

3. Tentukan bilangan asli terbesar n sehingga n3 + 100 kelipatan n + 10.

4. Diberikan f (x) = ax2 + bx + c, dengan a, b, dan c adalah bilangan bulat. Jika 3|f (x) untuksetiap bilangan bulat x, tunjukkan bahwa 3|a, 3|b, dan 3|c.

5. Buktikan bahwa n(n+1)(2n+1)6 merupakan bilangan bulat untuk sebarang bilangan bulat n.

6. Buktikan pernyataan-pernyataan di bawah ini:

(a) hasil kali 2 bilangan bulat yang berurutan selalu habis dibagi 2,

(b) hasil kali 3 bilangan bulat yang berurutan selalu habis dibagi 6,

(c) hasil kali n bilangan bulat yang berurutan selalu habis dibagi n!.

7. Buktikan bahwa gcd (a, b) = gcd (3a + 5b, 11a + 18b) .

8. Buktikan beberapa pernyataan berikut

(a) jika gcd (a, b) dan c|a maka gcd (b, c) = 1

(b) jika gcd (a, b) = 1 dan c| (a + b) maka gcd (a, c) = gcd (b, c) = 1

(c) jika gcd (a, b) = 1 maka gcd(a2, b2

)= 1.

9. Misalkan an = k2n, dengan k bilangan asli. Tunjukkan bahwa jika m 6= n maka gcd (am, an) =

1 untuk k genap, dan gcd (am, an = 2) untuk k ganjil.

10. Jika a,m, n bilangan asli, a > 1 dan gcd (m,n) = d, tunjukkan bahwa

[am − 1, an − 1] =(am − 1) (an − 1)

ad − 1

(petunjuk: tunjukkan bahwa gcd (am − 1, an − 1) = ad − 1).

3 Bilangan Prima

3.1 Pengertian bilangan prima

Pada pembahasan keterbagian, kita kenal istilah a membagi b. Nah, untuk selanjutnya pernyataana membagi b dapat kita katakan a faktor dari b.

Definisi 1.Bilangan bulat positif p dikatakan bilangan prima jika p mempunyai tepat dua faktor positif

yaitu 1 dan p sendiri.

Definisi 2.Bilangan bulat positif n dikatakan bilangan komposit jika n mempunyai lebih dari 2 faktor

positif.

23

Page 27: Materi-OSN-MatematikaSMA

Definisi 2 di atas juga dapat kita katakan bahwa n adalah bilangan komposit jika terdapatbilangan bulat positif a, b > 1 sehingga n = ab.

Contoh 1.Bilangan-bilangan 2, 3, 5, 7, ... merupakan bilangan prima. Bilangan 4, 6, 8, ... merupakan

bilangan komposit.Teorema 1Banyak bilangan prima adalah tak hingga.Bukti:Andaikan hanya ada sejumlah berhingga bilangan prima, sebut saja p1, p2, ..., pn dengan p1 <

p2 < ... < pn. Bentuk N = p1p2...pn + 1, jelas bahwa N > pn. Perhatikan bahwa untuk setiapk = 1, 2, ..., n haruslah pk tidak membagi N, karena jika pk membagi N maka pk|1 yang jelas tidakmungkin. Dengan demikian N prima atau terbagi oleh bilangan prima yang lebih dari pn. Hal inikontradiksi dengan asumsi kita. Jadi banyak bilangan prima adalah tak hingga.

Contoh 2.Tentukan semua bilangan prima yang berbentuk n3 + 1 untuk suatu bilangan asli n.Jawab:Perhatikan bahwa n3 +1 = (n + 1)

(n2 − n + 1

), dengan demikian salah satu faktor yaitu n+1

atau n2 − n + 1 harus sama dengan 1. Jika n + 1 = 1 maka n = 0 (tidak memenuhi), dan jikan2 − n + 1 = 1 maka n = 0 atau n = 1. Untuk n = 1 kita peroleh n3 + 1 = 2. Jadi bilangan primayang berbentuk n3 + 1 hanyalah 2.

Contoh 3.Tunjukkan bahwa untuk setiap bilangan prima p selalu berlaku 6|

(p2 − 1

).

Jawab:Setiap bilangan prima lebih p > 3 selalu dapat kita nyatakan sebagai 6k + 1 atau 6k− 1. Akan

tetapi, apapun yang terjadi akan selalu kita peroleh p2 = 6m + 1 di mana m = 6k2 + 2k ataum = 6k2 − 2k. Dengan demikian 6|

(p2 − 1

).

Akibat 1.

1. Jika p prima, maka untuk sebarang bilangan asli n berlaku p|n atau gcd (p, n) = 1.

2. Jika p prima dan p|ab untuk suatu bilangan bulat a dan b, maka p|a atau p|b.

Bukti:

1. Jika p|n, maka tidak ada yang perlu dibuktikan. Asumsikan p - n. Misalkan d = gcd (p, n) ,yang berarti d|p dan d|n. Karena p prima maka d = 1 atau d = p. Dari asumsi kita punyap - n, akibatnya d 6= n .Dengan kata lain d = 1 dan kita selesai.

2. Jika p|a, maka kita selesai. Asumsikan p - a. Menurut sifat (1) kita punya gcd (p, a) = 1,dengan demikian terdapat bilangan bulat x dan y sehingga px + ay = 1. Dengan mengalikankedua ruas dengan b akan kita peroleh pbx + aby = b atau setara dengan p (bx) = b − aby.Akan tetapi p|ab akibatnya p|b.�

Akibat 2.Jika p prima dan n, m sebarang bilangan asli dengan p|nm maka p|n.Bukti langsung dapat dilihat dari akibat 1 bagian 2.

24

Page 28: Materi-OSN-MatematikaSMA

3.2 Faktorisasi Prima

Bilangan prima merupakan bilangan yang lebih sederhana daripada bilangan komposit karena bi-langan prima hanya mempunyai 2 faktor positif yang berbeda. Oleh karena itu setiap bilanganasli akan kita bawa ke dalam perkalian bilangan-bilangan prima berpangkat yang disebut denganfaktorisasi prima. Berikut penjelasannya.

Teorema 2Setiap bilangan asli n > 1 dapat dinyatakan secara tunggal sebagai

n = pa11 pa2

2 ...pakk

dengan k suatu bilangan asli, p1 < p2 < ... < pk bilangan-bilangan prima berbeda , dan ai ≥ 1untuk setiap i = 1, 2, 3, ..., k.

Bukti:Jika n = p prima, maka kita selesai. Asumsikan n komposit. Misalkan p1 adalah bilangan

prima terkecil yang membagi n, maka n = p1n1 untuk suatu bilangan asli n1. Jika n1 prima, makakita selesai. Jika n1 tidak prima kita dapat menemukan bilangan prima terkecil yang membagi n1.Jika bilangan prima tersebut sama dengan p1 maka n = p2

1n2 untuk suatu bilangan asli n2, jikabilangan prima tersebut tidak sama dengan p1 sebut saja p2 maka = p1p2n2. Demikian seterusnya,sampai kita peroleh nm = 1 (mengapa ini dijamin?). Jadi

n = pa11 pa2

2 ...pakk

Sekarang akan kita buktikan ketunggalannya. Misalkan n = pa11 pa2

2 ...pakk = qb1

1 qb22 ...qbm

m denganp1 < p2 < ... < pk dan q1 < q2 < ... < qm bilangan-bilangan prima. Jika terdapat pi yang tidaksama dengan qt untuk setiap t = 1, 2, ...,m maka pi - qb1

1 qb22 ...qbm

m yang berarti pi - n dan ini tidakmungkin. Jadi, untuk setiap pi pasti terdapat t ∈ {1, 2, ...,m} sehingga pi = qt. Dengan cara yangsama dapat dibuktikan juga bahwa untuk setiap qi pasti terdapat t ∈ {1, 2, ..., k} sehingga qi = pt.Dengan demikian kita punya k = m. Dan karena p1 < p2 < ... < pk dan q1 < q2 < ... < qk makapi = qi untuk setiap i = 1, 2, ..., k atau dengan kata lain

pa11 pa2

2 ...pakk = pb1

1 pb22 ...pbk

k

Jika terdapat i sehingga ai > bi maka dengan membagi masing-masing ruas dengan pbii akan be-

rakibat pi membagi ruas kiri tetapi tidak membagi ruas kanan yang jelas tidak mungkin. Demikianjuga jika terdapat i sehingga ai < bi maka dengan membagi kedua ruas dengan pai

i akan berakibatpi tidak membagi ruas kiri tetapi pi membagi ruas kanan yang jelas tidak mungkin. Jadi kita haruspunya ai = bi untuk setiap i = 1, 2, ..., k. Dan kita selesai membuktikan ketunggalannya. �

keterangan: Tidak hanya bilangan asli saja yang dapat kita tulis dalam bentuk perkalianfaktor-faktor prima. Secara umum, untuk sebarang bilangan bulat n 6= 0 selalu dapat ditulisdalam bentuk n = upa1

1 pa22 ...pak

k dengan u = ±1, k bilangan asli, dan p1, p2, ..., pk bilangan prima.

Faktorisasi prima ini, akan memudahkan kita dalam menganalisa suatu bilangan bulat. Per-hatikan bahwa jika bentuk faktorisasi prima dari n adalah

n = pa11 pa2

2 ...pakk

maka kita mengetahui beberapa hal sebagai berikut:

25

Page 29: Materi-OSN-MatematikaSMA

1. n mempunyai k faktor prima yaitu p1, p2, ..., pk.

2. Banyak faktor positif dari n adalah (1 + a1) (1 + a2) ... (1 + an) (mengapa?).

Salah satu penggunaan faktorisasi prima adalah mencari FPB dan KPK dua bilangan bulat.Jika kita sudah mendapatkan faktorisasi prima dari dua bilangan bulat, sebut saja a dan b, makapastilah faktorisasi dari keduanya dapat dinyatakan sebagai

a = ps11 ps2

2 ...pskk dan b = pt1

1 pt22 ...ptk

k

dengan pi prima dan si, ti ≥ 0 untuk i = 1, 2, ..., k. Dengan faktorisasi prima ini, dapat dipahamibahwa

gcd (a, b) = pm11 pm2

2 ...pmkk dan [a, b] = pM1

1 pM22 ...pMk

k

di mana mi = min {ai, bi} dan Mi =maks{ai, bi} untuk setiap i = 1, 2, ..., k.

Untuk lebih jelasnya, mari kita simak beberapa contoh di bawah ini:

Contoh 4Tentukan FPB dan KPK dari 56 dan 2008.Jawab:Akan kita selesaikan dengan faktorisasi prima, perhatikan bahwa 112 = 24.7 dan 2008 = 23.251.

Dengan demikian gcd (56, 2008) = 23 = 8 dan [112, 208] =(24

)(7) (251) = 28 112.

Contoh 5Tentukan jumlahan dari semua faktor positif dari 5.000.000.Jawab:Tentu kita dapat mendaftar semua faktor positif dari 5.000.000 kemudian menjumlahkannya,

namun akan butuh waktu yang sangat lama. Nah, sekarang perhatikan bahwa faktorisasi primadari 5.000.000 adalah 26.57. Dengan demikian, setiap faktor positifnya berbentuk 2a.5b dengana = 0, 1, 2, ..., 6 dan b = 0, 1, 2, ..., 7. Sehingga jumlahan dari semuanya adalah

∑a,b

(2a.5b

)=(∑6

a=02a

) (∑7

b=03b

)=

(27 − 1

) (38−1

2

).

Contoh 6

Jika m adalah bilangan asli sehingga√

m merupakan bilangan rasional, maka tunjukkan bahwam merupakan kuadrat suatu bilangan asli.

Jawab:Misalkan

√m = a

b dengan a dan b bilangan asli (mengapa?). Jika b = 1, maka jelas bahwam = a2. Asumsikan b > 1, akibatnya a > 1. Kita dapat tulis faktorisasi prima dari a dan b yaitu a =ps11 ps2

2 ...pskk dan b = pt1

1 pt22 ...ptk

k . Sekarang perhatikan bahwa m = a2

b2= p2s1−2t1

1 p2s2−2t22 ...p2sk−2sk

k =(ps1−t11 ps2−t2

2 ...psk−skk

)2. Karena m bilangan asli dan p1, p2, ..., pk bilangan prima maka bilangan

yang berada dalam tanda kurung merupakan bilangan asli, dan kita selesai.

3.3 Soal-soal Latihan

1. Tentukan semua bilangan asli n yang menyebabkan n4 + 4 merupakan bilangan prima.

2. Tunjukkan n merupakan bilangan komposit jika dan hanya jika n terbagi oleh bilangan primap dengan p ≤

√n.

26

Page 30: Materi-OSN-MatematikaSMA

3. Tunjukkan bahwa ada tak hingga bilangan prima yang berbentuk 4n+3 untuk suatu bilanganasli n.

4. Apakah terdapat bilangan asli n sehingga 6n + 5 merupakan jumlahan dari dua bilanganprima?

5. Tentukan semua bilangan prima p sehingga 4p2 + 1 dan 6p2 + 1 juga merupakan bilanganprima. (soal OSN I, Yogyakarta, 2002).

6. Diberikan p > 3 adalah bilangan prima. Jika

1 +12

+13

+ ... +1

p− 1=

a

b

dengan a dan b bilangan bulat dan gcd (a, b) = 1, maka tunjukkan bahwa p membagi a.

7. Tunjukkan bahwa jika a, b, c, d ∈ N dan ab = cd, maka bilangan an + bn + cn + dn merupakanbilangan komposit untuk setiap bilangan asli n.

8. Jika faktorisasi prima dari n adalah n = pa11 pa2

2 ...pakk , maka tunjukkan bahwa

(a) Hasil kali semua faktor positif dari n adalah nm2 dengan m = (1 + a1) (1 + a2) ... (1 + ak) ,

(b) Jumlahan semua faktor positif daro n adalah(∑a1

i=0 pi1

) (∑a2i=0 pi

2

)...

(∑aki=0 pi

k

).

9. Tunjukkan bahwa n merupakan kuadrat sempurna jika dan hanya jika n mempunyai sejumlahganjil faktor positif.

10. Terdapat 2008 pintu berjajar diberi nomor 1 sampai 1008 dan semuanya dalam keadaantertutup. Sekelompok anak, Pi dengan 1 ≤ i ≤ 2008 berjalan melalui jajaran pintu tersebut.Masing-masing anak mengubah kondisi pintu nomor k jka dan hanya jika i membagi k, jikapintunya tertutup diubah menjadi terbuka dan sebaliknya. Cari banyak pintu yang terbukasetelah semua anak melewati jajaran pintu tersebut.

4 Persamaan dan Sistem Persamaan dalam Bilangan Bulat

Dalam menyelesaikan persamaan dan sistem persamaan dalam bilangan bulat (sering juga disebutpersamaan Diophantine) tentu akan lebih mudah, karena kita hanya dibatasi penyelesaian dalambilangan bulat. Sebagai contoh jika kita akan mencari pasangan bilangan real (x, y) yang memenuhixy = 2, tentu akan ada tak hingga banyaknya yaitu semua pasangan bilangan real

(x, 2

x

)untuk

setiap bilangan real tak nol x, pasti merupakan solusi xy = 2. Akan tetapi jika kita akan mencaripasangan bilangan bulat (x, y) yang memenuhi persamaan xy = 2, maka solusinya hanya ada 4yaitu (1, 2) , (2, 1) , (−1,−2) , dan (−2,−1) . Mengapa demikian? Untuk lebih jelasnya simak uraianberikut:

27

Page 31: Materi-OSN-MatematikaSMA

4.1 Persamaan Diophantine Linear

Persamaan ini adalah persamaan yang paling sederhana, karena kita bisa langsung mencari solusiumumnya.

Definisi 1Misalkan a, b, dan c adalah bilangan-bilangan bulat. Persamaan Diophantine berbentuk ax +

by = c disebut Persamaan Diophantine linear dan setiap pasangan bilangan bulat (x, y) yangmemenuhi ax + by = c disebut solusi.

Teorema 1Persamaan Diophantine ax + by = c mempunyai solusi jika dan hanya jika gcd (a, b) |c.Bukti:=⇒)Diketahui persamaan ax+ by = c mempunyai solusi, artinya ada bilangan bulat x0 dan y0 yang

memenuhi ax0 + by0 = c. Andaikan gcd (a, b) tidak membagi c. Perhatikan bahwa ruas kiri terbagioleh gcd (a, b) tetapi ruas kanan tidak terbagi oleh gcd (a, b) yang jelas ini tidak mungkin. Jadiharuslan gcd (a, b) membagi c.

⇐=)Diketahui gcd (a, b) |c, artinya terdapat bilangan bulat k sehingga c = k gcd (a, b) .Menurut identitas Benzout terdapat bilangan bulat m dan n yang memenuhi am + bn =

gcd (a, b) . Dengan mengambil x = km dan y = kn kita akan punya

ax + by = akm + bkn = k (am + bn) = k (gcd (a, b)) = c

yang berarti persamaan ax + by = c mempunyai solusi yaitu (km, kn) .

Contoh 2Hitung banyak bilangan bulat 1 ≤ n ≤ 100 yang dapat dinyatakan dalam bentuka 6x + 8y

untuk suatu bilangan bulat x dan y.Jawab:Perhatikan bahwa gcd (6, 8) = 2. Oleh karena itu menurut teorema di atas, hanya bilangan yang

terbagi oleh 2 yang dapat dinyatakan dalam bentuk 6x + 8y untuk suatu bilangan bulat x dan y.Dalam hal ini, 1 ≤ n ≤ 100 yang terbagi oleh 2 ada tepat 50 bilangan.

Teorema 2Jika Persamaan Diophantine ax + by = c mempunyai solusi (x0, y0) maka persamaan tersebut

mempunyai tak hinga banyaknya solusi dan setiap solusinya berbentuk

x (k) = x0 + kb

gcd (a, b)dan y (k) = y0 − k

a

gcd (a, b)

untuk sebarang bilangan bulat k.Bukti:Diketahui (x0, y0) solusi dari ax + by = c, artinya ax0 + by0 = c. Jika (x (k) , y (k)) kita substi-

tusikan ke persamaan akan kita peroleh

ax (k) + by (k) = ax0 + kab

gcd (a, b)+ by0 − k

ab

gcd (a, b)= ax0 + by0 = c

28

Page 32: Materi-OSN-MatematikaSMA

yang berarti (x (k) , y (k)) juga merupakan solusi.Nah, untuk bukti bahwa solusi persamaan ax +by = c hanyalah (x (k) , y (k)) akan kita bahas setelah kita membahas kongruensi bilangan bulat(pada modul ini tidak dibahas).

Pertanyaannya adalah bagaimana cara kita menentukan solusi awal (x0, y0) ini? Ingat kembaliwaktu kita membahas Algoritma Euclide. Kita bisa mencari bilangan bulat m dan n sehinggaam + bn = gcd (a, b) . Karena persamaan ax + by = c punya solusi jika dan hanya jika gcd (a, b) |cmaka terdapat k sehingga c = k gcd (a, b) . Dengan demikian kita bisa mengambil solusi awalx0 = km dan y0 = kn. (Pada umumnya, bukan pekerjaan yang sulit untuk mencari salah satusolusi dari persamaan ax + by = c).

Contoh 3Tentukan semua solusi dari Persamaan Diophantine linear 6x + 8y = 12.Jawab:Kita punya gcd (6, 8) = 2 dan 2|12, yang berarti persamaan ini punya solusi. Mudah dipahami

bahwa salah satu solusinya adalah (2, 0) . Dengan demikian solusi umumnya adalah x (k) = 2 + 4kdan y (k) = −3k untuk sebarang bilangan bulat k.

Contoh 4Ada berapa banyak pasangan bilangan asli (x, y) yang memenuhi persamaan 4x + 6y = 48?Jawab:Untuk menyelesaikan soal ini kita selesaikan seperti biasa yaitu kita cari solusi umumnya, selan-

jutnya kita batasi nilai k agar solusinya merupakan bilangan asli. Salah satu solusi dari persamaanini adalah (12, 0) , sehingga solusi umumnya adalah x = 12 + 3k dan y = −2k untuk sebarangbilangan bulat k. Sekarang akan kita batasi nilai k sehingga x, y > 0. Dari 12+3k > 0 kita perolehk > −4 dan dari −2k > 0 kita peroleh k < 0, atau dengan kata lain kita peroleh −4 < k < 0.Akan tetapi, karena k bilangan bulat maka −3 ≤ k ≤ −1. Dengan demikian ada tepat 3 pasanganbilangan asli (x, y) yang memenuhi 4x + 6y = 48.

4.2 Persamaan Diophantine Non Linear

Persamaan ini sangat banyak bentuknya, kita tidak mungkin mengkarakteristik satu persatu. Disini kita hanya memaparkan dengan beberapa teknik melalui contoh-contoh soal:

Contoh 5 (teknik pemfaktoran)Tentukan solusi bulat dari persamaan xy = 2x− y.Jawab:Perhatikan bahwa soal di atas ekivalen dengan xy− 2x + y = 0, dengan menambahkan masing-

masing ruas dengan −2, akan diperoleh xy − 2x + y − 2 = −2 dan ini dapat difaktorkan menjadi(x + 1) (y − 2) = −2. Karena x dan y bilangan bulat, maka demikian juga dengan x + 1 dan y− 1.Dengan demikian, ada 4 kejadian yang mungkin

(i). x + 1 = −1 dan y − 2 = 2. Dari sini diperoleh solusi x = −2, y = 4,

(ii). x + 1 = 1 dan y − 2 = −2. Dari sini diperoleh solusi x = 0, y = 0,

(iii). x + 1 = 2 dan y − 2 = −1. Dari sini diperoleh solusi x = 1, y = 1,

(iv). x + 1 = −2 dan y − 2 = 1. Dari sini diperoleh solusi x = −3, y = 3.

29

Page 33: Materi-OSN-MatematikaSMA

Mudah dicek bahwa keempat pasang solusi memenuhi persamaan yang diberikan. Jadi semuasolusinya dapat kita nyatakan dalam pasangan (−2, 4) , (0, 0) , (1, 1) , dan (−3, 3) .

Cotoh 6 (teknik pembatasan)Tentukan bilangan asli a, b, c sehingga 1

a + 1b + 1

c = 1.Jawab:Perhatikan bahwa persamaan di atas simetri, artinya jika a kita tukar dengan b dan b kita

tukar dengan a persamaan tidak berubah. Sehingga dapat kita asumsikan a ≥ b ≥ c. Akibatnnya1 = 1

a + 1b + 1

c ≤1c + 1

c + 1c = 3

c ⇔ c ≤ 3. Dari sini kita hanya cukup mengecek untuk c = 1, 2, 3.

• c = 1, kita substitusikan ke persamaan awal akan kita peroleh 1a + 1

b = 0, dan ini tidak punyapenyelesaian bilangan asli.

• c = 2, kita substitusikan ke persamaan awal akan kita peroleh 1a + 1

b = 12

Karena a ≥ b, maka kita peroleh 12 = 1

a + 1b ≤

2b ⇔ b ≤ 4. dan juga kita punya b ≥ c = 2.

. • b = 2 tidak ada a yang memenuhi.. b = 3, kita peroleh a = 6.

. b = 4, kita peroleh a = 4.

• c = 3, kita substitusikan ke persamaan awal akan kita peroleh 1a + 1

b = 23 .

Karena a ≥ b, maka kita peroleh 23 = 1

a + 1b ≤

2b ⇔ b ≤ 3. dan juga kita punya b ≥ c = 3

jadi b = 3, sehingga kita peroleh a = 3.Kita peroleh pasangan solusi (6, 3, 2) , (4, 4, 2) , (3, 3, 3). Perhatikan bahwa awalnya kita asum-

sikan a ≥ b ≥ c, padahal bisa saja a ≥ c ≥ b atau yang lainnya. Tetapi karena persamaan-nya simetris, maka solusi yang lainnya tingal diubah urutannya. Jadi semua solusinya adalah(6, 3, 2) , (6, 2, 3) , (3, 2, 6) ,

(3, 6, 2) , (2, 3, 6) , (2, 6, 3) , (4, 4, 2) , (4, 2, 4) , (2, 4, 4) , dan (3, 3, 3) .Contoh 7 (teknik keterbagian)Tunjukkan bahwa tidak ada bilangan bulat x dan y yang memenuhi pesamaan

(x + 1)2 + (x + 2)2 + ... + (x + 99)2 = y2

Jawab:Andaikan terdapat bilangan bulat x dan y yang demikian. Kita jabarkan yang ruas kiri, yakni:

(x + 1)2 + (x + 2)2 + ... + (x + 99)2 = 99x2 + 99 (100) x +99.100.199

6= 99

(x2 + 100x

)+ 33.50.199

dengan demikian kita punya 99(x2 + 100x

)+ 33.50.199 = y2. Perhatikan bahwa ruas kiri habis

dibagi 3, akibatnya ruas kanan juga habis dibagi 3. Akan tetapi, karena ruas kanan merupakankuadrat sempurna maka ruas kanan juga kan dibagi 9. Tentu saja ruas kiri juga havis dibagi 9,akibatnya 33.50.199 habis dibagi 9 yang jelas ini tidak mungkin.

Contoh 8 (teknik parameter)Tunjukkan bahwa persamaan x2 + y2 = x3 mempunyai tak hingga banyaknya solusi asli.Jawab:Persamaan di atas dapat kita tulis sebagai y2 = x3 − x2 atau ekivalen dengan y2 = x2 (x− 1) .

Oleh karena itu, agar persamaan tersebut punya solusi, kita harus punya x− 1 merupakan kuadratsempurna. Dengan mengambil x = n2 +1 dan y = n

(n2 + 1

)untuk sebarang bilangan asli n, maka

mudah ditunjukkan bahwa pasangan(n2 + 1, n

(n2 + 1

))merupakan solusi yang banyaknya jelas

ada tak hingga.

30

Page 34: Materi-OSN-MatematikaSMA

4.3 Sistem Persamaan Diophantine

Telah kita bahas beberapa jenis dan contoh Persamaan Diophantine serta cara menyelesaikannya.Nah, untuk menyelesaikan sistem persamaan Diophantine kita dapat membawa ke dalam bentukPersamaan Diophantine seperti yang telah kita kenal. Pada dasarnya, kita akan mencari solusibilangan asli yang memenuhi semua persamaan yang diberikan secara simultan.

4.4 Soal-soal Latihan

1. Ada berapa banyak pasangan bilangan asli (x, y) yang memenuhi persamaan 2x+3y = 1000?

2. Banyak pasangan bilangan asli (x, y) yang memnuhi persamaan 1x −

1y = 1

3 adalah...

3. Banyaknya bilangan asli n sehingga 3n + 81 merupakan kuadrat sempurna adalah..

4. Bilangan bulat positif terkecil n sehingga 31 membagi 5n + n adalah...

5. Tentukan semua bilangan asli a, b, dan c yang menyebabkan 1a + 1

b + 1c merupakan bilangan

asli.

6. Tentukan semua pasangan bilangan bulat non negatif yang memenuhi persamaan

(xy − 7)2 = x2 + y2

7. Diketahui x, y, z, dan n adalah bilangan-bilangan asli yang memenuhi

xn + yn = zn

Tunjukkan bahwa x, y, dan z semuanya lebih besar dari n.

8. Tentukan semua bilangan real a sehingga persamaan kuadrat x2 + ax + 6a = 0 mempunyaidua solusi yang keduanya bulat.

9. Tentukan semua pasangan bilangan bulat (x, y) yang memenuhi

x4 + x3 + x2 + x + 1 = y2

10. Carilah semua bilangan prima p sehingga sistem persamaan{p + 1 = 2x2

p2 + 1 = 2y2

mempunyai solusi bulat.

5 Fungsi Tangga

Dalam dunia jual beli, biasanya penjual ingin menjual harganya semahal mungkin, dan sebaliknyapembeli ingin membeli barang yang ia inginkan semurah mungkin. Bahkah, kadang-kadang jikaharganya tidak bulat ribuan misalnya 7.300 maka penjual ingin dibayar 7.500, sebaliknya pembeliingin membayar dengan harga yang dibulatkan ke bawah yaitu 7.000. Nah, dalam ilmu matematikakita akan mengenal fungsi yang digunakan oleh penjual dan pembeli di atas. Fungsi yang akankita pelajari jika digambarkan pada bidang kartesius akan berbentuk seperti tangga. Ada 3 macamfungsi tangga yang akan kita bahas dalam bab ini, yaitu fungsi floor (pembulatan ke bawah), fungsiceiling (pembulatan ke atas), dan fungsi bulat (pembulatan ke bilangan bulat yang terdekat).

31

Page 35: Materi-OSN-MatematikaSMA

5.1 Fungsi floor

Fungsi floor disebut juga fungsi pembulatan ke bawah, yakni dengan mengambil bagian bulatnya.Untuk sebarang bilangan real x,nilai fungsi floor dari x kita tulis dengan bxc .

Definisi 1Misalkan x adalah sebarang bilangan real. Nilai fungsi floor x kita tulis dengan bxc merupakan

bilangan bulat terbesar yang kurang dari atau sama dengan x.Contoh:b3, 14c = 3, b−2, 5c = −3,

⌊√2⌋

= 1, dan lain sebagainya.

Definisi 2Untuk sebarang bilangan real x, notasi {x} menyatakan bagian pecahan dari x.Secara matematika, definisi di atas dapat kita tuliskan

{x} = x− bxc

Dari sini jelas bahwa untuk sebarang bilangan real x berlaku 0 ≤ {x} < 1.Contoh:{3, 14} = 0, 14; {−2, 5} = 0, 5;

{√2}

= 0, 41..., dan lain sebagainya.

Langsung dari definisi, kita dapat menurunkan beberapa sifat sebagai berikut:

1. Untuk sebarang bilangan real x selalu berlaku x− 1 < bxc ≤ x,

2. bxc = x jika dan hanya jika x ∈ Z,

3. bx + kc = bxc+ k untuk sebarang bilangan bulat k,

4. bxc+ byc ≤ bx + yc untuk setiap x, y ∈ R,

5. bxyc ≤ bxc byc untuk setiap x, y ∈ R.

Sifat 1, 2, dan 3 trivial. Di sini kita hanya akan membuktikan sifat 4, sedangkan untuk sifat 5buktinya hampir sama dengan pembuktian sifat 4 dan diserahkan kepada pembaca sebagai latihan.

Bukti sifat 4.Tulis x = bxc+ {x} dan y = byc+ {y} , akan kita peroleh

bx + yc = bbxc+ byc+ {x}+ {y}c = bxc+ byc+ b{x}+ {y}c ≥ bxc+ byc

dan kita selesai. �

Contoh 1Untuk sebarang bilangan real x, tunjukkan bahwa

⌊x + 1

2

⌋+ bxc = b2xc .

Bukti:Tulis x = bxc + {x} dengan 0 ≤ {x} < 1. Kita bagi 2 kasus, yaitu jika 0 ≤ {x} < 1

2 dan12 ≤ {x} < 1.

(i). untuk 0 ≤ {x} < 12 , kita punya⌊

x +12

⌋+ bxc =

⌊bxc+ {x}+

12

⌋+ bxc = 2 bxc+

⌊{x}+

12

⌋= 2 bxc

danb2xc = b2 bxc+ 2 {x}c = 2 bxc+ b2 {x}c = 2 bxc

yang jelas bahwa ruas kiri sama dengan ruas kanan.

32

Page 36: Materi-OSN-MatematikaSMA

(ii). untuk 12 ≤ {x} < 1, kita punya⌊

x +12

⌋+ bxc =

⌊bxc+ {x}+

12

⌋+ bxc = 2 bxc+

⌊{x}+

12

⌋= 2 bxc+ 1

danb2xc = b2 bxc+ 2 {x}c = 2 bxc+ b2 {x}c = 2 bxc+ 1

yang juga jelas bahwa ruas kiri sama dengan ruas kanan.

Perhatikan bahwa jika n dan a adalah sebarang bilangan asli, mudah dipahami bahwa bn/acmerupakan banyaknya bilangan kelipatan a di antara 1, 2, ..., n. Fakta ini sederhana tetapi dapatkita gunakan untuk menyelesaikan beberapa permasalahan seperti pada contoh berikut.

Contoh 2Tentukan banyak anggota himpunan {1, 2, ..., 100} yang habis dibagi 2 atau 3.Jawab:Banyak bilangan kelipatan 2 ada b100/2c = 50, banyak bilangan kelipatan 3 ada b100/3c = 33,

dan banyak bilangan kelipatan 2 dan 3 ada b100/6c = 16. Oleh karena itu, dengan prinsip inklusieksklusi kita peroleh bahwa banyak kelipatan 2 atau 3 ada b100/2c + b100/3c − b100/6c = 50 +33− 16 = 67 bilangan.

Contoh 3Hitung banyak nol di sebelah kanan tanpa terputus dari 31!.Jawab:Angka nol di sebelah kanan tanpa terputus pada 31! akan dihasilkan pada saat kita menga-

likankelipatan 10, dan faktor 10 ini didapat dari kelipatan genap dan kelipatan 5. Karena banyakkelipatan 2 lebih banyak daripada banyak kelipatan 5, maka kita cukup menghitung kelipatan 5.Perhatikan juga bahwa mungkin bilangan kelipatan 5 yang kita kalikan tersebut juga merupakankelipatan 25 dan jika dikalikan dengan bilangan kelipatan 4 akan menghasilkan bilangan kelipatan100 yang akan menambah nol di sebelah kanan sebanyak 2, akan tetapi sebanyak 1 nol telah telahkita masukkan saat kita menghitung kelipatan 5. Dengan demikian, banyak nol di sebelah kanantanpa terputus dari 31! adalah b31/5c+ b31/25c = 6 + 1 = 7 nol.

5.2 Fungsi ceiling

Fungsi floor disebut juga fungsi pembulatan ke atas. Untuk sebarang bilangan real x,nilai fungsiceiling dari x kita tulis dengan dxe .

Definisi 3Misalkan x adalah sebarang bilangan real. Nilai fungsi ceiling x kita tulis dengan dxe merupakan

bilangan bulat terkecil yang lebih dari atau sama dengan x.Contoh:d3, 14e = 4, d−2, 5e = −2,

⌈√2⌉

= 2, dan lain sebagainya.

5.3 Fungsi bulat

Fungsibulat disebut juga fungsi pembulatan ke bilangan bulat terdekat. Untuk sebarang bilanganreal x,nilai fungsi bulat dari x kita tulis dengan [x] .

Definisi 4

33

Page 37: Materi-OSN-MatematikaSMA

Misalkan x adalah sebarang bilangan real. Nilai fungsi bulat x kita tulis dengan [x] merupakanbilangan bulat terdekat dengan x. Jika x = k+ 1

2 untuk suatu bilangan bulat k, maka kita definisikan[x] = k + 1.

Contoh:[3, 14] = 3, [−2, 5] = −2,

[√2]

= 1, dan lain sebagainya

Dari definisi di atas, kita dapat menurunkan beberapa sifat berikut:

1. bxc ≤ [x] ≤ dxe untuk setiap bilangan real x.

2. dxe = x jika dan hanya jika [x] = x,

3. dx + ke = dxe+ k dan [x + k] = [x] + k untuk sebarang bilangan bulat k,

4. dxe+ dye ≥ dx + ye untuk setiap x, y ∈ R,

5. dxye ≤ dxe dye untuk setiap x, y ∈ R.

Contoh 4Tunjukkan bahwa untuk setiap bilangan real x selalu berlaku

⌊x + 1

2

⌋= [x]

Jawab:Tulis x = bxc+{x} , dengan 0 ≤ {x} < 1. Jika 0 ≤ {x} < 1

2 maka⌊x + 1

2

⌋= bxc dan [x] = bxc .

Jika 12 ≤ {x} < 1 maka

⌊x + 1

2

⌋= bxc+ 1 dan [x] = bxc+ 1.

Contoh 5Tentukan semua bilangan real x yang memenuhi persamaan⌈

x2⌉

+⌊x2

⌋= 2003

(soal OSN SMA tahun 2003, bidang matematika)Jawab:Perhatikan bahwa jika x2 bulat maka

⌈x2

⌉=

⌊x2

⌋yang akan berakibat x2 = 2003

2 yang jelasbukan bilangan bulat. Jadi, x2 harus bukan bilangan bulat. Akibatnya

⌈x2

⌉=

⌊x2

⌋+ 1, sehingga

2⌊x2

⌋+ 1 = 2003 yang selanjutnya kita peroleh

⌊x2

⌋= 1001. Dari sini kita simpulkan 1001 ≤

x2 < 1002 yang kemudian kita dapatkan penyelesaiannya adalah −√

1002 < x ≤ −√

1001 atau√1001 ≤ x <

√1002.

5.4 Soal-soal Latihan

1. Buktikan atau beri contoh penyangkal dari pernyataan-pernyataan berikut

(a) [x + y] ≤ [x] + [y] untuk setiap x, y ∈ R,

(b) [xy] ≤ [x] [y] untuk setiap x, y ∈ R,

(c)⌊bxcm

⌋=

⌊xm

⌋untuk setiap x ∈ R dan bilangan bulat m.

2. Misalkan x dan y adalah bilangan real yang memenuhi bx + yc = bxc+ byc dan b−x− yc =b−xc+ b−yc . Buktikan bahwa x atau y merupakan bilangan bulat.

34

Page 38: Materi-OSN-MatematikaSMA

3. Untuk setiap bilangan real x didefinisikan bxc sebagai bilangan bulat terbesar yang kurangdari atau sama dengan x. Misalkan a dan b adalah bilangan real positif sehingga a bac = 17dan b bbc = 11. Tentukan nilai dari a− b.

4. Hitung1[1]

+1[2]

+1[3]

+ ... +1

[1000]

5. Cari semua bilangan asli n sehingga banyak nol di sebelah kanan tanpa terputus dari n! tepatada 10 nol.

6. Diketahui S = {1, 2, ..., 100} . Hitung banyaknya anggota S yang merupakan kelipatan 2 ataukelipatan 3 tetapi bukan kelipatan 5.

7. Tentukan semua bilangan real x yang memenuhi persamaan bxc dxe = x2

8. Hitung banyak bilangan real x dengan 1 ≤ x ≤ 100 yang memenuhi persamaan

x2 −⌊x2

⌋= (x− bxc)2

9. Tunjukkan bahwa untuk setiap m bilangan asli,⌊

m2

3

⌋+

⌊(m+1)2

3

⌋+

⌊(m+2)2

3

⌋merupakan

kuadrat sempurna.

10. Misalkan an =⌊(

1 +√

2)n

⌋. Tunjukkan bahwa an ganjil jika n genap, dan genap jika n

ganjil.

35

Page 39: Materi-OSN-MatematikaSMA

Geometry

Fajar Yuliawan

Sebelum kita mulai pembahasan bab geometri ini, ada beberapa hal yang perludiketahui oleh pembaca. Geometri yang dibahas di sini tidak akan mencakup se-bagaian besar geometri dasar yang pernah diajarkan di sekolah menengah. Hal-haldasar yang tidak dibahas di sini adalah hubungan antara garis dengan titik, hubun-gan antara garis dengan garis, kesebangunan dan kekongruenan segitiga, hubungangaris dengan lingkaran (berpotongan dan bersinggungan), pengenalan trigonometridan bangun-bangun ruang sederhana. Materi-materi tersebut dapat didapatkansendiri di sebagian besar buku-buku matematika untuk sekolah menengah.

Pembahasan geometri disini selanjutnya lebih ditekankan pada pembuktian-pembuktian beberapa teorema maupun fakta-fakta dalam geometri. Teoremayang dibahas di bab geometri ini memang tidak banyak, namun diharapkan dapatmengenalkan pembaca pada pembuktian fakta-fakta geometri dan menjadi dasarpembuktian-pembuktian soal-soal olimpiade. Seringkali untuk membuktikan se-buah soal olimpiade, yang diperlukan adalah ide-ide pada pembuktian suatu teo-rema, bukan teorema yang bersangkutan. Oleh karena itu, pembaca diharapkandapat memahami setiap bukti yang ada pada setiap teorema. Dalam hal ini,pembaca diharapkan dapat mengetahui dua hal, yaitu kebenaran setiap langkahpembuktian dan tujuan setiap langkah tersebut. Jika ada keraguan mengenai su-atu langkah pembuktian, pembaca seharusnya menanyakan hal ini kepada temanmaupun guru matematika di sekolah.

1 Titik-titik dan Garis-garis yang Terhubung de-

ngan Segitiga

Pertama, kita buat beberapa konvensi ntuk mempermudah penulisan. Untuk se-tiap segitiga ABC, kita gunakan notasi untuk panjang, yaitu a = BC, b = CA,c = AB, s = (a+b+c)/2 dan notasi untuk sudut, yaitu A = ∠BAC, B = ∠ABC,dan C = ∠BCA. Kemudian untuk luas, kita menggunakan notasi [XY Z] untukmenyatakan luas segitiga XY Z. Lebih umum, untuk setiap poligon P , maka [P ]menyatakan luas poligon tersebut.

36

Page 40: Materi-OSN-MatematikaSMA

1.1 Konkurensi dan Kolinearitas pada Segitiga: TeoremaCeva dan Teorema Menelaos

Kita mulai dengan segmen-segmen garis yang menghubungkan titik sudut suatusegitiga dengan sebuah titik yang terletak pada sisi di depan titik sudut tersebut.Segmen garis seperti itu disebut sebagai cevian (diambil dari nama Giovanni Ceva,seorang matematikawan Italia yang pertama kali menyinggung masalah konkurensitiga buah cevian).

Teorema Ceva. Misalkan ABC sebuah segitiga dan D, E, F tiga titik yangberturut-turut terletak pada sisi-sisi BC, CA, AB. Maka garis-garis AD, BE, CFkonkuren jika dan hanya jika

BD

DC

CE

EA

AF

FB= 1.

Bukti. Teorema di atas membutuhkan pembuktian ”dua arah”, yaitu: jikaAD, BE, CF konkuren, maka kesamaan di atas berlaku dan jika kesamaan berlaku,maka AD, BE, CF konkuren.

F'

P P'

D

EF

D

EF

B

A

C B

A

C

Gambar 1.

Pertama, kita buktikan dulu bahwa jika AD, BE, CF konkuren maka ke-samaan yang diberikan berlaku. Misalkan P adalah titik perpotongan ketiga garisAD, BE, CF . Perhatikan dua identitas berikut:

BD

DC=

[ABD]

[ACD]dan

BD

DC=

[PBD]

[PCD],

yang diperoleh dari fakta bahwa jika dua buah segitiga memiliki ”tinggi” yangsama, maka perbandingan luasnya sama dengan perbandingan ”alas”-nya. Daridua identitas tersebut, kemudian kita peroleh

BD

DC=

[ABD]− [PBD]

[ACD]− [PCD]=

[APB]

[CPA](Mengapa?).

37

Page 41: Materi-OSN-MatematikaSMA

Dengan cara yang sama, kita peroleh

CE

EA=

[BPC]

[APB]dan

AF

FB=

[CPA]

[BPC].

Jadi,BD

DC

CE

EA

AF

FB=

[APB]

[CPA]

[BPC]

[APB]

[CPA]

[BPC]= 1.

Sekarang misalkan kesamaan di atas berlaku. Akan dibuktikan bahwa AD,BE, CF berpotongan di satu titik. Untuk membuktikan hal ini, kita meng-gunakan teknik titik bayangan (phantom point). Perhatikan Gambar 1 sebelahkanan. Misalkan cevian AD dan BE berpotongan di titik P ′ dan garis CP ′ mem-otong sisi AB di titik F ′. Kita cukup membuktikan bahwa F ′ = F , atau dengankata lain, kedua titik tersebut berimpit (Mengapa?). Untuk membuktikan hal ini,pertama perhatikan bahwa tiga cevian AD, BE, CF ′ konkuren (bertemu di titikF ′). Dengan demikian, kita punya

BD

DC

CE

EA

AF ′

F ′B= 1 =

BD

DC

CE

EA

AF

FB,

sehinggaAF ′

F ′B=

AF

FB.

Dari sini kita simpulkan F = F ′ (Mengapa?) dan kita selesai.

Pada kasus-kasus tertentu, teorema Ceva di atas lebih mudah digunakan dalambentuk trigonometri berikut:

Akibat (”Trig Ceva”). Misalkan ABC sebuah segitiga dan P, Q,R tigatitik yang berturut-turut terletak pada sisi-sisi BC, CA, AB. Maka garis-garisAP, BQ, CR konkuren jika dan hanya jika

sin ∠CAP

sin ∠APB

sin ∠ABQ

sin ∠QBC

sin ∠BCR

sin ∠RCA= 1.

Akibat di atas dapat dibuktikan dengan mudah dengan menggunakan aturansinus dan teorema Ceva atau secara langsung dengan menggunakan aturan sinuspada beberapa segitiga. Bukti selengkapnya diserahkan kepada pembaca

Tiga buah titik dikatakan kolinear jika terletak pada satu garis. Kriteria kolin-earitas tiga titik yang berada pada ketiga sisi-sisi segitiga diberikan oleh Menelaos.

38

Page 42: Materi-OSN-MatematikaSMA

Teorema Menelaos. Misalkan ABC sebuah segitiga dan D, E, F tiga titikpada garis-garis BC, CA, AB (D, E, F bisa terletak pada perpanjangan sisi-sisisegitiga ABC). Maka D, E, F kolinear jika dan hanya jika

BD

DC

CE

EA

AF

FB= 1.

Bukti. Pertama, kita buktikan bahwa jika D, E, F kolinear, maka kesamaanyang diberikan berlaku. Buat garis tegak lurus dari tiga A, B, C terhadap garisyang melalui D, E, F dan misalkan P , Q, R adalah ketiga kaki tegaklurusnya(perhatikan Gambar 2).

R

P

Q

D

E

F

C

A

B

Gambar 2.

Kita punya tiga kesamaan berikut yang dapat diperoleh dengan meninjaukesebangunan-kesebangunan beberapa segitiga:

BD

DC=

BQ

CR,

CE

EA=

CR

AP, dan

AF

FB=

AP

BQ.

Dengan mengalikan ketiga kesamaan tersebut, kita peroleh

BD

DC

CE

EA

AF

FB=

BQ

CR

CR

AP

AP

BQ= 1.

Bukti untuk arah yang satunya (yaitu jika kesamaan berlaku, maka ketigatitik kolinear) dapat dibuktikan dengan menggunakan titik bayangan, sama sepertibukti teorema Ceva. Hal ini dilakukan dengan memisalkan F ′ sebagai perpotongangaris-garis AB dan DE lalu membuktikan bahwa F = F ′. Bukti selengkapnyadiserahkan kepada pembaca.

39

Page 43: Materi-OSN-MatematikaSMA

1.2 Panjang Cevian: Teorema Stewart

Panjang cevian dapat dihitung dengan menggunakan teorema berikut:

Teorema Stewart. Misalkan AX adalah sebuah cevian dengan panjang pyang membagi sisi BC menjadi dua segmen, yaitu BX dengan panjang m danXC dengan panjang n. Maka berlaku

a(p2 + mn) = b2m + c2n.

Bukti.

nm

p bc

X CB

A

Gambar 3.

Dengan menggunakan aturan cosinus pada segitiga ABX dan ACX, kita per-oleh

cos ∠AXB =p2 + m2 − c2

2pmdan cos ∠AXC =

p2 + n2 − b2

2pn.

Karena ∠AXB = 180◦ − ∠AXC, maka cos ∠AXB = − cos ∠AXC atau setaradengan cos ∠AXB + cos ∠AXC = 0. Dengan demikian, kita punya

p2 + m2 − c2

2pm+

p2 + n2 − b2

2pn= 0,

yang setara dengan

n(p2 + m2 − c2) + m(p2 + n2 − b2) = 0,

atau setara juga dengan

(m + n)(p2 + mn) = b2m + c2n

dan setara dengan kesamaan yang diinginkan, karena m + n = a.

Dengan teorema di atas, panjang cevian AX dapat dihitung secara langsung,yaitu

p =

√b2m + c2n

a−mn.

40

Page 44: Materi-OSN-MatematikaSMA

1.3 Titik-titik dan Garis-garis Istimewa pada Segitiga

Pada sebuah segitiga, terdapat banyak titik dan garis istimewa, namun dalamsubbab ini, hanya akan dijelaskan beberapa diantaranya, yaitu garis berat, titikberat, garis tinggi, titik tinggi, garis bagi sudut, titik pusat lingkaran dalam, garissumbu dan titik pusat lingkaran luar.

1.3.1 Garis Berat (Median) dan Titik Berat (Centroid)

Kita mulai dengan definisi garis berat. Garis berat sebuah segitiga adalah garisyang melalui titik sudut segitiga tersebut dan titik tengah sisi di depannya. Dengandemikian, setiap segitiga akan memiliki tiga garis berat. Dengan menggunakanteorema Ceva, teorema berikut dapat dibuktikan dengan mudah:

Teorema. Misalkan ABC sebuah segitiga dan A′, B′, C ′ titik-titik tengah sisi-sisi BC, CA, AB. Maka garis-garis berat AA′, BB′, CC ′ konkuren.

G

A'

C' B'

C

A

B

Gambar 4.

Titik potong ketiga garis berat sebuah segitiga kemudian dinamakan titik berat.Karena berpotongan di satu titik, maka ketiga garis berat sebuah segitiga akanmembagi segitiga tersebut menjadi enam bagian. Selain itu, setiap garis berat akanmembagi garis berat lainnya menjadi dua bagian. Kita punya teorema berikut:

Teorema. Misalkan AA′, BB′, CC ′ adalah tiga garis berat segitiga ABC yangberpotongan di titik berat G. Maka keenam segitiga AGB′, AGC ′, BGA′, BGC ′,CGA′ dan CGB′ memiliki luas yang sama.

Bukti. Perhatikan kembali Gambar 4. Karena A′, B′, C ′ berturut-turutadalah titik-titik tengah BC, CA, AB, kita punya bahwa [BGA′] = [CGA′],

41

Page 45: Materi-OSN-MatematikaSMA

[AGB′] = [CGB′], dan [AGC ′] = [BGC ′]. Kita juga punya bahwa [ABA′] =[ACA′], sehingga

2[AGC ′] = [ABG] = [ABA′]− [BGA′]

= [ACA′]− [CGA′] = [ACG]

= 2[AGB′],

sehingga [AGC ′] = [AGB′]. Dengan cara yang sama diperoleh [BGA′] = [AGB′] =[AGC ′], dan kesimpulan mengikuti.

Teorema. Misalkan AA′, BB′, CC ′ adalah tiga garis berat segitiga ABC yangberpotongan di titik berat G. Maka

AG

GA′ =BG

GB′ =CG

GC ′ = 2.

Bukti. Kita gunakan lagi teorema sebelumnya. Kita punya bahwa

AG

GA′ =[ABG]

[GBA′]=

[AGC ′] + [BGC ′]

[GBA′]= 2,

karena [AGC ′] = [BGC ′] = [GBA′]. Kesamaan lain dapat diperoleh dengan carayang sama.

Panjang garis berat sendiri dapat dihitung dengan mudah menggunakan teo-rema Stewart.

1.3.2 Garis Bagi Sudut (Bisector)

Garis yang membagi sebuah sudut segitiga menjadi dua bagian yang sama besardinamakan garis bagi sudut. Pertama, kita punya teorema berikut:

Teorema. Misalkan AA′, BB′, CC ′ adalah garis-garis bagi sudut segitiga ABC(dengan demikian, ∠A′AB = ∠A′AC = 1

2A, dan seterusnya). Maka

A′B

A′C=

c

b,

B′C

B′A=

a

c, dan

C ′A

C ′B=

b

a.

Bukti.

42

Page 46: Materi-OSN-MatematikaSMA

A' CB

A

Gambar 5.

Dengan aturan sinus pada segitiga-segitiga AA′B dan AA′C, kita peroleh

A′B

AB=

sin ∠A′AB

sin ∠AA′Bdan

A′C

AC=

sin ∠A′AC

sin ∠AA′C.

Karena ∠A′AB = ∠A′AC dan sin ∠AA′B = 180◦−sin ∠AA′C, maka sin ∠A′AB =sin ∠A′AC dan sin ∠AA′B = sin ∠AA′C. Akibatnya,

A′B

AB=

sin ∠A′AB

sin ∠AA′B=

sin ∠A′AC

sin ∠AA′C=

A′C

AC,

sehingga kita perolehA′B

A′C=

AB

AC=

c

b.

Dua kesamaan sisanya dapat dibuktikan dengan cara yang sama.

Dengan menggunakan teorema di atas dan teorema Ceva atau langsung denganmenggunakan Trig Ceva, kita peroleh teorema berikut:

Teorema. Misalkan AA′, BB′, CC ′ adalah tiga garis bagi sudut segitiga ABC.Maka ketiga garis tersebut konkuren.

Selain itu, karena perbandingan A′B/A′C dan A′B + A′C dapat dinyatakandalam panjang sisi-sisi a, b, c, maka panjang A′B dan A′C juga dapat dinyatakandalam a, b, c. Selanjutnya, dengan teorema Stewart, panjang garis bagi sudut AA′

juga dapat dihitung. Perhitungan ini diserahkan kepada pembaca sebagai latihan.

1.3.3 Garis Tinggi (Altitude) dan Titik Tinggi (Orthocenter)

Garis tinggi sebuah segitiga adalah garis yang melalui titik sudut sebuah segitigadan tegak lurus dengan sisi di depannya. Kita punya teorema berikut

43

Page 47: Materi-OSN-MatematikaSMA

Teorema. Misalkan AA′, BB′, CC ′ garis-garis tinggi sebuah segitiga (dengandemikian, AA′ tegak lurus BC, dan seterusnya). Maka ketiga garis tersebutkonkuren.

Bukti.

A'

B'

C'

CB

A

Gambar 6.

Kasus dimana ABC segitiga siku-siku trivial (sangat jelas dan tidak ada yangperlu dibuktikan), karena ketiga garis tinggi akan berpotongan pada titik sudutsiku-siku segitiga ABC). Jadi kita cukup meninjau dimana ABC bukan segitigasiku-siku.

Sekarang akan kita buktikan teorema tersebut untuk kasus dimana ABC se-gitiga lancip. Kita punya bahwa

AA′

BA′ = tan B danAA′

CA′ = tan C.

Dengan demikian,BA′

CA′ =tan C

tan B.

Dengan cara yang sama, diperoleh

CB′

AB′ =tan A

tan Cdan

AC ′

BC ′ =tan B

tan A.

Akibatnya,BA′

A′C

CB′

B′A

AC ′

C ′B=

tan C

tan B

tan A

tan C

tan B

tan A= 1,

sehingga AA′, BB′, CC ′ konkuren.

Kasus terakhir dimana segitiga ABC tumpul diserahkan kepada pembaca. Satuhal yang perlu diperhatikan dalam pembuktian kasus tersebut adalah bahwa titikperpotongan ketiga garis tinggi terletak di luar segitiga ABC.

44

Page 48: Materi-OSN-MatematikaSMA

Titik potong ketiga ketiga garis tinggi sebuah segitiga selanjutnya disebut se-bagai titik tinggi (orthocenter) segitiga. Kemudian, sama seperti pada garis bagisudut, panjang BA′ dan CA′ dapat dinyatakan dalam panjang sisi-sisi a, b, c danfungsi trigonometri sudut-sudut A, B, C. Dengan menyatakan fungsi trigonometrisudut dalam panjang sisi (misalnya dengan aturan cosinus), panjang BA′ dan CA′

dapat dinyatakan dalam panjang sisi-sisi a, b, c. Selanjutnya, teorema Stewartdapat digunakan untuk menghitung panjang garis tinggi AA′.

1.3.4 Garis Sumbu (Perpendicular Bisector)

Garis sumbu sebuah segitiga adalah garis yang melalui titik tengah sebuah sisi dantegak lurus terhadap sisi tersebut. Kita punya teorema berikut:

Teorema. Misalkan lA, lB, lC adalah garis-garis sumbu segitiga ABC yangberturut-turut tegak lurus terhadap sisi-sisi BC, CA, AB (dengan demikian, lAmelalui titik tengan BC, dan seterusnya). Maka lA, lB, lC konkuren.

Bukti. Kita tidak dapat menggunakan teorema Ceva untuk membuktikanteorema ini karena garis-garis sumbu sebuah segitiga bukan merupakan cevian.Untuk membuktikan teorema ini, kita cukup membuktikan bahwa titik potongdua buah garis terletak pada garis yang ketiga (Mengapa?).

lA

lB

O

B'

A'B

A

C

Gambar 7.

Misalkan A′, B′, C ′ berturut-turut adalah titik-titik tengah sisi-sisi BC, CA,AB. Misalkan juga O adalah perpotongan garis lA dan lB. Sekarang akan dibuk-tikan bahwa O terletak pada garis lC . Pertama, tinjau segitiga OA′B dan OA′C.Dengan teorema Pythagoras pada kedua segitiga tersebut dan karena A′ adalahtitik tengah sisi BC, kita punya

OB =√

A′B2 + A′O2 =√

A′C2 + A′O2 = OC.

Dengan cara yang sama, kita punya bahwa OC = OA. Jadi, kita punya OA = OB,sehingga OAB segitiga sama kaki. Oleh karena itu, garis tinggi segitiga OAB dari

45

Page 49: Materi-OSN-MatematikaSMA

titik O akan memotong titik tengah AB, atau dengan kata lain O terletak padagaris yang melalui titik tengah AB dan tegak lurus garis AB, yaitu garis lC . Halini melengkapkan pembuktikan.

1.4 Lingkaran Dalam (incircle) dan Lingkaran Luar Segit-iga (circumcircle)

Lingkaran dalam segitiga adalah lingkaran yang menyinggung ketiga sisi segitigadari dalam dan lingkaran luar segitiga adalah lingkaran yang melalui ketiga titik-titik sudut segitiga. Kita punya beberapa teorema berikut mengenai titik pusatlingkaran dalam dan luar serta panjang jari-jarinya.

Teorema. Titik perpotongan ketiga garis bagi sudut segitiga ABC adalahtitik pusat lingkaran dalam segitiga ABC dan panjang jari-jarinya sama dengan[ABC]/s.

Bukti.

P

R

Q

I

A'

B'

C'

CB

A

Gambar 8.

Misalkan AA′, BB′ dan CC ′ adalah ketiga garis bagi sudut segitiga ABC yangberpotongan di titik I. Misalkan juga P , Q, R berturut-turut adalah kaki tegaklurus titik I pada sisi-sisi BC, CA, AB, atau dengan kata lain, P , Q, R terletakpada sisi-sisi BC, CA, AB sedemikian hingga IP tegak lurus BC, IQ tegak lurusCA dan IR tegak lurus AB. Karena ∠QAI = ∠CAA′ = ∠BAA′ = ∠RAI dan∠AQI = 90◦ = ∠ARI maka kedua segitiga siku-siku AQI dan ARI sebangun.Kemudian karena sisi miring kedua segitiga siku-siku tersebut berimpit (sehinggasama panjang), maka kedua segitiga tersebut sebangun. Oleh karenanya, kitapunya bahwa IQ = IR. Dengan cara yang sama, diperoleh juga bahwa IP = IQ.Jadi kita peroleh IP = IQ = IR. Akibatnya, lingkaran dengan pusat I danberjari-jari IP = IQ = IR menyinggung sisi-sisi BC, CA, AB. Lingkaran tersebut

46

Page 50: Materi-OSN-MatematikaSMA

kemudian disebut sebagai lingkaran dalam segitiga (incenter) ABC dan jari-jarinya(IP = IQ = IR) disebut sebagai jari-jari lingkaran dalam segitiga (inradius) ABC.

Sekarang misalkan r menyatakan panjang jari-jari lingkaran dalam segitigaABC. Karena segitiga AQI dan ARI kongruen maka AQ = AR dan

[AQIR] = 2× [AQI] = 2× 1

2· AQ · IQ =

1

2(AQ + AR) · r.

Dengan cara yang sama, diperoleh

[BRIP ] =1

2(BR + BP ) · r dan [CPIQ] =

1

2(CP + CQ) · r.

Akibatnya,

[ABC] = [AQIR] + [BRIP ] + [CPIQ]

=1

2(AQ + AR) · r +

1

2(BR + BP ) · r +

1

2(CP + CQ) · r

=1

2(AR + BR + BP + CP + CQ + AQ) · r

=1

2(AB + BC + CA) · r

= sr,

atau setara dengan r = [ABC]/s.

Teorema. Titik perpotongan ketiga garis sumbu segitiga ABC adalah titikpusat lingkaran luar segitiga ABC dan panjang jari-jarinya sama dengan abc/4[ABC].

Bukti.

lB

lA

C'

O

A'

B'

C

A

B

47

Page 51: Materi-OSN-MatematikaSMA

Gambar 9.

Misalkan lA, lB, lC adalah ketiga garis sumbu segitiga ABC dan O adalah per-potongan ketiga garis tersebut. Dengan cara yang sama seperti bukti teoremakonkurensi garis sumbu, kita peroleh bahwa OA = OB = OC. Akibatnya, Oadalah titik pusat lingkaran yang melalui ketiga titik sudut segitiga ABC (yaitulingkaran yang berpusat di O dengan panjang jari-jari OA = OB = OC). Ling-karan tersebut selanjutnya dinamakan lingkaran luar segitiga (circumcircle) ABCdan titik pusatnya disebut titik pusat lingkaran luar segitiga (circumcenter) ABC.

Sekarang misalkan AL adalah sebuah garis tinggi segitiga ABC, sehingga

AL

AB= sin B dan [ABC] =

1

2×BC × AL.

Jadi,

[ABC] =1

2×BC × AL =

1

2×BC × AB sin B =

1

2ac sin B.

Selanjutnya dengan aturan sinus, kita punya bahwa

b

sin B= 2R,

sehingga

[ABC] =1

2ac sin B =

1

2ac× b

2R=

abc

4R,

atau setara dengan

R =abc

4[ABC].

2 Lingkaran dan Beberapa Sifatnya

2.1 Kuasa Titik Terhadap Lingkaran (Power of a Pointwith Respect to a Circle)

Pembahasan pertama di subbab ini adalah tentang hubungan sebuah titik dengansebuah lingkaran. Di sini kita mengenal konsep kuasa sebuah titik terhadap sebuahlingkaran.

Teorema. Diberikan sebuah lingkaran dan sebuah titik P . Buat sebuah garisyang memotong lingkaran di dua titik A dan B (jika garis tersebut menyinggunglingkaran, maka A = B). Maka hasil kali PA×PB bernilai tetap, tidak tergantungoleh garis yang dibuat.

48

Page 52: Materi-OSN-MatematikaSMA

Bukti. Buat garis lain yang melalui titik P dan memotong lingkaran di duatitik C dan D. Sekarang perhatikan bahwa ∠PBC = ∠PDA dan ∠BPC =∠DPA.

PP

C

B

A

D

B

AD

C

Gambar 10.

Dengan demikian, kedua segitiga PBC dan PDA sebangun, sehingga

PB

PD=

PC

PA,

yang setara dengan PA× PB = PC × PD.

Sekarang kita gunakan notasi yang sama dengan notasi pada teorema di atas.Misalkan O dan r berturut-turut adalah tiik pusat dan panjang jari-jari lingkarantersebut. Tinjau garis yang melalui O dan P yang memotong lingkaran di duatitik R dan S. Nilai

(OP − r)(OP + r) = OP 2 − r2

disebut sebagai kuasa titik P terhadap lingkaran tersebut. Jika P berada di dalamlingkaran, maka kuasanya negatif, jika P di luar maka kuasanya positif dan jika Pterletak pada lingkaran, kuasanya nol.

S

R

S

R

P

B

P

B

A

O

A

O

Gambar 11.

49

Page 53: Materi-OSN-MatematikaSMA

Terkait dengan teorema sebelumnya, jika P berada di luar atau pada lingkaran,maka

PA× PB = PR× PS = (OP − r)(OP + r) = OP 2 − r2

sama dengan kuasa P terhadap lingkaran tersebut dan jika P di dalam lingkaran,maka

PA× PB = PR× PS = (r −OP )(r + OP ) = r2 −OP 2

sama dengan −1 kali kuasa P terhadap lingkaran tersebut.

2.2 Jarak Titik Pusat Lingkaran Dalam dan Luar Segitiga:Teorema Euler

Teorema kuasa titik terhadap lingkaran di atas dapat digunakan untuk membuk-tikan salah satu teorema penting dalam geometri: Teorema Euler.

Teorema Euler. Misalkan I dan O berturut-turut adalah titik-titik pusatlingkaran dalam dan luar segitiga ABC. Jika r dan R berturut-turut menyatakanpanjang jari-jari lingkaran dalam dan luar segitiga ABC, maka

OI2 = R2 − 2rR.

Bukti.

α

β

αα

β

N

I

L

M

CB

A

O

Gambar 12.

Misalkan N adalah kaki tegak lurus I pada sisi CA, sehingga IN = r dangaris bagi sudut AI memotong lingkaran luar segitiga ABC di titik L. Karena∠BAL = ∠CAL, maka L adalah titik tengah busur BC yang tidak memuat A.

50

Page 54: Materi-OSN-MatematikaSMA

Sekarang misalkan garis LO memotong lingkaran luar segitiga ABC lagi di titikM , sehingga LM adalah diameter lingkaran tersebut yang tegak lurus denganBC (Mengapa?). Sekarang misalkan α = 1

2A dan β = 1

2B. Kita punya bahwa

∠BML = ∠BAL = α,sehingga

LB

2R=

LB

LM= sin α =

IN

IA=

r

IA,

sehingga LB = 2R sin α dan IA = r/ sin α.

Kita juga punya bahwa ∠LBC = ∠LAC = α, sehingga

∠LBI = α + β = 180◦ − ∠AIB = ∠LIB,

sehingga LBI adalah segitiga sama kaki, sehingga LB = LI. Sekarang karena Iberada di dalam lingkaran luar segitiga ABC, maka LI× IA sama dengan −1 kalikuasa I terhadap lingkaran luar segitiga ABC, yaitu R2 − OI2. Di sisi lain, kitapunya bahwa

LI × IA = LB × IA = 2R sin α× r

sin α= 2rR,

sehingga R2 −OI2 = 2rR yang setara dengan kesamaan yang ingin dibuktikan.

Sebagai akibat dari teorema tersebut, kita punya ketaksamaan berikut

Akibat. Jika R dan r berturut-turut menyatakan panjang jari-jari lingkaranluar dan dalam suatu segitiga, maka

R ≥ 2r.

2.3 Segiempat Talibusur dan Beberapa Sifatnya

Segiempat talibusur adalah segiempat yang keempat titik sudutnya terletak padasatu lingkaran. Berikutnya akan dijelaskan beberapa kriteria dan sifat segiempattalibusur.

2.3.1 Beberapa Kriteria Segiempat Talibusur

Misalkan ABCD sebuah segiempat talibusur dan O adalah titik pusat lingkaranluarnya. Kita punya bahwa ∠ADB = ∠ACB

(= 1

2∠AOB

)dan ∠ADC+∠ABC =

180◦. Konvers dari pernyataan tersebut ternyata berlaku. Kita punya teoremaberikut:

51

Page 55: Materi-OSN-MatematikaSMA

Teorema. Jika ABCD sebuah segiempat konveks (yaitu kedua diagonalnyaterletak di dalam segiempat) dan ∠ADB = ∠ACB, maka ABCD segiempat tal-ibusur.

Bukti. Kita menggunakan teknik titik bayangan lagi. Misalkan lingkaranluar segitiga ABC memotong garis BD di titik D′ (dalam hal ini, D′ adalah titikbayangan dari titik D). Selanjutnya, kita cukup membuktikan bahwa D′ = D(Mengapa?).

D

D'

C

B

A

D

D'

C

B

A

Gambar 13.

Perhatikan bahwa ABCD′ adalah segiempat talibusur, sehingga kita punya∠ACB = ∠AD′B.Dan karena ∠ACB = ∠ADB, maka ∠ADB = ∠AD′B. Aki-batnya, D = D′ (Mengapa?) dan kita selesai.

Teorema. Jika ABCD sebuah segiempat dan ∠ADC + ∠ABC = 180◦, makaABCD segiempat talibusur.

Teorema tersebut dapat dibuktikan dengan menggunakan konsep titik bayan-gan dan pembuktian ini diserahkan kepada pembaca.

Selain dua kriteria dasar di atas, konsep titik bayangan dan teorema kuasatitik terhadap lingkaran dapat digunakan untuk membuktian kriteria segiempattalibusur berikut.

Teorema. Misalkan ABCD sebuah segiempat talibusur. Misalkan juga ADdan BC berpotongan di E dan AB dan CD berpotongan di F . Jika salah satudari dua kesamaan berikut:

EA× EC = EB × ED atau FA× FB = FC × FD,

maka ABCD adalah segiempat talibusur.

52

Page 56: Materi-OSN-MatematikaSMA

2.3.2 Teorema Ptolemeus dan Brahmagupta

Kita akhiri subbab ini dengan dua teorema tentang segiempat talibusur. Teoremapertama menyebutkan hubungan antara panjang diagonal dan panjang sisi-sisisegiempat talibusur dan teorema berikutnya tentang hubungan luas segiempattalibusur dengan panjang sisi-sisi nya.

Teorema. Misalkan ABCD sebuah segiempat talibusur. Maka

AB × CD + BC × AD = AC ×BD.

Bukti.

E

B C

D

A

Gambar 14.

Misalkan E titik pada segmen AC sehingga ∠ABE = ∠DBC. Karena ∠BAE =∠BAC = ∠BDC, maka segitiga ABE sebangun dengan segitiga DBC, sehingga

AE

AB=

CD

BD

atau setara dengan AE ×BD = AB × CD. Sekarang perhatikan bahwa

∠CBE = ∠ABC − ∠ABE = ∠ABC − ∠DBC = ∠DBA

dan ∠BCE = ∠BCA = ∠BDA. Dengan demikian, segitiga BCE sebangundengan segitiga BDA, sehingga

CE

BC=

AD

BD

atau setara dengan CE ·BD = BC · AD. Jadi,

AC ×BD = AE ×BD + CE ×BD = AB × CD + BC × AD.

53

Page 57: Materi-OSN-MatematikaSMA

Teorema berikut ini diperoleh oleh Brahmagupta, seorang matematikawan In-dia pada abad ketujuh A.D.

Teorema. Misalkan ABCD sebuah segiempat talibusur dengan panjang sisi-sisi AB = a, BC = b, CD = c, DA = d dan s = (a + b + c + d)/2. Maka luassegiempat talibusur tersebut adalah

[ABCD] =√

(s− a)(s− b)(s− c)(s− d)

Bukti.

π − α

α

D

BA

C

Gambar 15.

Salah satu cara termudah membuktikan teorema di atas adalah menggunakantrigonometri. Misalkan ∠ABC = α, sehingga ∠ADC = 180◦−α. Sekarang denganaturan cosinus pada segitiga ABC dan ADC, kita peroleh

AC2 = a2 + b2 − 2ab cos α dan AC2 = c2 + d2 − 2cd cos(180◦ − α).

Karena cos(180◦ − α) = − cos α, dari dua kesamaan di atas kita peroleh

a2 + b2 − 2ab cos α = c2 + d2 + 2cd cos α,

yang setara dengan

2(ab + cd) cos α = a2 + b2 − c2 − d2,

sehingga

cos2 α =

(a2 + b2 − c2 − d2

2(ab + cd)

)2

Sekarang perhatikan bahwa

[ABC] =1

2ab sin α dan [ACD] =

1

2cd sin(180◦ − α) =

1

2cd sin α,

54

Page 58: Materi-OSN-MatematikaSMA

sehingga

[ABCD] = [ABC] + [ACD] =1

2(ab + cd) sin α.

Dengan demikian

[ABCD]2 =1

4(ab + cd)2 sin2 α =

1

4(ab + cd)2(1− cos2 α)

=1

4(ab + cd)2

(1−

(a2 + b2 − c2 − d2

2(ab + cd)

)2)

=1

4(ab + cd)2 − 1

16(a2 + b2 − c2 − d2)2.

Di sisi lain, dengan menjabarkan kedua ruas, kita juga punya bahwa

(s− a)(s− b)(s− c)(s− d) =1

4(ab + cd)2 − 1

16(a2 + b2 − c2 − d2)2.

Kesimpulan selanjutnya mengikuti.

Sebuah kasus khusus dimana titik D berimpit dengan titik A atau C akanmenghasilkan teorema Heron.

Teorema. Misalkan ABC sebuah segitiga dengan panjang sisi-sisi a, b, c dans = (a + b + c)/2. Maka

[ABC] =√

s(s− a)(s− b)(s− c).

3 Segiempat

Tidak banyak hal yang bisa dibahas dalam segiempat secara umum. Pada beber-apa kasus, sebuah segiempat lebih mudah dipandang sebagai dua buah segitigayang memiliki satu sisi persekutuan.

3.1 Jajaran Genjang Varignon

Pembahasan pertama ini adalah mengenai bangun datar yang dibentuk oleh titik-titik tengah keempat sisi sebuah segiempat. Kita memiliki teorema berikut:

Teorema. Jika K, L, M , N adalah titik-titik tengah sisi-sisi AB, BC, CD,DA pada segiempat ABCD, maka KLMN adalah jajaran genjang. Jajaran gen-jang yang terbentuk tersebut disebut sebagai jajaran genjang Varignon. Lebihjauh, luas jajaran genjang Varignon sama dengan setengah luas segiempatnya.

55

Page 59: Materi-OSN-MatematikaSMA

Bukti.

K

L

M

N

D

C

B

A

Gambar 16.

Perhatikan segitiga ABC. Karena K dan L berturut-turut adalah titik-titiktengah AB dan BC, maka KL tegak lurus AC. Dengan cara yang sama, kitapunya bahwa MN juga tegak lurus dengan AC. Akibatnya, KL sejajar denganAC. Dengan cara yang sama juga, kita peroleh KN sejajar LM . Kesimpulanmengikuti dengan mudah.

Untuk luasnya, kita punya bahwa

[KLMN ] = [ABCD]− [AKN ]− [BKL]− [CLM ]− [DMN ]

= [ABCD]− 1

4[ABD]− 1

4[ABC]− 1

4[BCD]− 1

4[CDA]

= [ABCD]− 1

4[ABCD]− 1

4[ABCD]

=1

2[ABCD].

3.2 Lingkaran Dalam Segiempat

Tidak semua segiempat memiliki lingkaran dalam, yaitu suatu lingkaran di dalamsegiempat yang menyinggung keempat sisi segiempat. Teorema berikut mem-berikan sifat sebuah segiempat yang memiliki lingkaran dalam

Teorema. Misalkan ABCD adalah segiempat konveks yang memiliki lingkarandalam. Maka AB + CD = BC + DA.

Bukti. Misalkan lingkaran dalam segiempat ABCD menyinggung sisi-sisi AB,BC, CD, DA berturut-turut di titik-titik K, L, M , N .

56

Page 60: Materi-OSN-MatematikaSMA

B

CD

A

L

M

N

K

Gambar 17.

Dengan demikian, kita punya

AK = AN , BK = BL, CL = CM , dan DM = DN

sehingga

AB + CD = AK + BK + CM + DM = AN + BL + CL + DN = BC + DA.

Konvers dari teorama di atas ternyata berlaku. Kita punya teorema berikut.

Teorema. Misalkan ABCD adalah segiempat konveks yang memenuhi AB +CD = BC + DA. Maka ABCD memiliki lingkaran dalam.

Bukti.

D

P

B

C

D'

A

L

M

K

Gambar 18.

57

Page 61: Materi-OSN-MatematikaSMA

Misalkan garis AB dan CD berpotongan di titik P dan Γ adalah lingkarandalam segitiga PBC. Misalkan juga garis melalui A yang menyinggung Γ mem-otong PC di titik D′ sehingga Γ juga merupakan lingkaran dalam segiempatABCD′. Selanjutnya, cukup dibuktikan bahwa D = D′.

Dengan menggunakan teorema sebelumnya, kita punya AB+CD′ = BC+D′Adan menggunakan kesamaan yang diberikan, kita punya

CD′ − CD = AB + CD′ − (AB + CD)

= BC + D′A− (BC + DA) = D′A−DA.

Jika CD′ > CD, maka DD′ = CD′ − CD = D′A − DA, sehingga D′A =DA+DD′ yang jelas tidak mungkin karena menurut ketaksamaan segitiga D′A <DA + DD′. Demikian juga halnya dengan CD′ < CD. Kita simpulkan bahwaCD′ = CD, sehingga D = D′ dan kita selesai.

4 Lampiran

4.1 Rumus-Rumus Trigonometri

Berikut diberikan rangkuman rumus-rumus trigonometri terutama untuk sinus dancosinus. Rumus untuk tangent dan cotangent dapat diturunkan dari rumus-rumustersebut. Pertama adalah rumus trigonometri untuk jumlah maupun selisih duasudut dan rumus untuk mengubah jumlah atau selisih sinus dan cosinus menjadihasil kali sinus dan cosinus.

cos(A + B) = cos A cos B − sin A sin B

cos(A−B) = cos A cos B + sin A sin B

sin(A + B) = sin A cos B + cos A sin B

sin(A−B) = sin A cos B − cos A sin B

sin A + sin B = 2 sin

(A + B

2

)cos

(A−B

2

)sin A− sin B = 2 cos

(A + B

2

)sin

(A−B

2

)cos A + cos B = 2 sin

(A + B

2

)cos

(A−B

2

)cos A− cos B = −2 sin

(A + B

2

)sin

(A−B

2

).

58

Page 62: Materi-OSN-MatematikaSMA

Selanjutnya adalah rumus trigonometri untuk dua kali sudut atau setengah kalisudut.

sin 2A = 2 sin A cos A

cos 2A = 2 cos2 A− 1 = 1− 2 sin2 A

tan 2A =2 tan A

1− tan2 A

sinA

2= ±

√1− cos A

2

cosA

2= ±

√1 + cos A

2.

4.2 Perluasan Aturan Sinus dan Aturan Cosinus

Teorema (Aturan Sinus). Misalkan ABC segitiga dengan panjang jari-jarilingkaran luar R. Maka

a

sin A=

b

sin B=

c

sin C= 2R.

Teorema (Aturan Cosinus). Misalkan ABC segitiga. Maka

a2 = b2 + c2 − 2bc cos A

b2 = c2 + a2 − 2ca cos B

c2 = a2 + b2 − 2ab cos C.

59

Page 63: Materi-OSN-MatematikaSMA

60

KOMBINATORIKA

Utari Wijayanti 1. Permutasi dan Kombinasi 1.1 Faktorial

Kita definisikan simbol ! (faktorial), sebagai berikut 0! = 1, dan untuk bilangan asli n1,

n! = 1 · 2 ··· n.

Perhatikan n! dibaca sebagai n faktorial. Contoh 1 Berdasarkan definisi di atas, kita dapatkan

1! = 1 2! = 1 · 2 = 2, 3! = 1 · 2 · 3 = 6, 4! = 1 · 2 · 3 · 4 = 24, 5! = 1 · 2 · 3 · 4 · 5 = 120.

Contoh 2 Perhatikan bahwa 7!4! 7.6.5.4!4! 210 2 !! 2 1 !! 2 1

2 !1 ! 2 !1 1 2 ! 11 1

1.2 Permutasi Tanpa Pengulangan

Definisi Misalkan x1, x2, . . . , xn merupakan n obyek berbeda. Suatu permutasi dari obyek-obyek ini merupakan penyusunan obyek-obyek tersebut dengan memperhatikan urutan. Untuk selanjutnya permutasi n obyek disebut sebagai n-permutasi. Contoh 3 Terdapat 24 permutasi untuk huruf-huruf pada kata MATH, yakni

MATH, MAHT, MTAH, MTHA, MHAT, MHTA,

AMTH, AMHT, ATMH, ATHM, AHMT, AHTM,

TAMH, TAHM, TMAH, TMHA, THAM, THMA,

HAMT, HATM, HMAT, HMTA, HTAM, HTMA.

Edited by Foxit Reader Copyright(C) by Foxit Corporation,2005-2009 For Evaluation Only.
Page 64: Materi-OSN-MatematikaSMA

61

Teorema Misalkan x1, x2, … , xn adalah n obyek yang berbeda. Maka banyaknya n − permutasi adalah n! permutasi.

Contoh 4 Banyaknya permutasi dari huruf-huruf dari kata RAMBUT adalah 6! = 720. Contoh 5 Sebuah lemari memuat 5 buku berbahasa Jerman, 7 buku berbahasa Spanyol, dan 8 buku berbahasa Indonesia. Diketahui bahwa tidak ada dua buku yang sama.

a. Berapa banyak penyusunan berbeda yang bisa dilakukan pada buku-buku ini? b. Berapa banyak penyusunan berbeda yang bisa dilakukan pada buku-buku ini jika

buku-buku dari masing-masing bahasa harus diletakkan saling berdekatan? c. Berapa banyak penyusunan berbeda yang bisa dilakukan pada buku-buku ini jika

semua buku berbahasa Indonesia harus saling berdekatan? d. Berapa banyak penyusunan berbeda yang bisa dilakukan pada buku-buku ini jika

tidak boleh ada dua buku berbahasa Indonesia diletakkan saling berdekatan?

Jawab: a. Perhatikan bahwa kita melakukan permutasi pada 5+7+8 = 20 buku. Maka banyaknya

kemungkinan penyusunan adalah 20!. b. Buku-buku dengan bahasa yang sama di’ikat’ sehingga saling berdekatan. Perhatikan

bahwa kita melakukan permutasi pada 3 bahasa, yakni dengan 3! cara. Kemudian, kita lakukan permutasi pada buku-buku berbasa Jerman dalam 5! cara, pada buku-buku berbahasa Spanyol dalam 7! cara, dan pada buku-buku berbahasa Indonesia dengan 8! cara. Jadi total banyaknya cara penyusunan buku adalah 3!5!7!8!.

c. Pertama-tama, kita atur dulu buku-buku berbahasa Jerman dan Spanyol. Perhatikan dengan mengatur 5 + 7 = 12 buku-buku ini, kita mendapatkan 13 ’ruang’, yakni ruang sebelum buku pertama, ruang di antara buku-buku, dan ruang setelah buku terakhir. Untuk memastikan bahwa buku-buku berbahasa Indonesia saling berdekatan, buku-buku ini kita ’ikat’ bersama-sama dan diletakkan pada salah satu ’ruang’. Perhatikan bahwa buku-buku berbahasa Indonesia dapat disusun dengan 8! cara, sedangkan buku-buku berbahasa Jerman dan Perancis dengan 12! cara. Sehingga total banyaknya cara adalah 13⋅8!12!.

d. Pertama-tama atur buku berbahasa Jerman dan Spanyol. Dengan mengatur 5 + 7 = 12 buku ini, diperoleh 13 ’ruang’. Untuk memastikan bahwa tidak ada buku-buku berbahasa Indonesia yang saling berdekatan, kita letakkan buku-buku tersebut ke dalam ’ruang-ruang’ ini. Buku pertama dapat ditempatkan ke salah satu dari 13 ruang. Buku kedua dapat ditempatkan ke sisa 12 ruang yang ada, buku ketiga dapat ditempatkan ke sisa 11 ruang yang ada, dan seterusnya, hingga buku ke 8 dapat ditempatkan ke sisa 6 ruang yang ada. Jadi total banyaknya pengaturan yang ada adalah 13 · 12 · 11 · 10 · 9 · 8 · 7 · 6 · 12!.

Latihan 6 Sebuah rak buku terdiri atas 3 novel Rusia, 4 novel Jerman, dan 5 novel Indonesia. Ada berapa banyak cara penyusunan novel-novel ini jika,

a. tidak ada batasan pengelompokkan novel b. semua novel Indonesia harus berdekatan c. tidak ada novel Indonesia yang saling berdekatan

Latihan 7 Berapa banyak kemungkinan permutasi untuk kata LEMARI? Berapakah banyak permutasi yang diawali dengan huruf R dan diakhiri dengan huruf E? Berapa banyak

Page 65: Materi-OSN-MatematikaSMA

62

permutasi jika P dan U selalu bersama-sama dengan urutan PU? Berapa banyak permutasi yang ada jika tidak ada huruf vokal (A, E, I) yang berdekatan? Latihan 8 Berapa banyak pengaturan yang bisa dilakukan pada huruf-huruf dari kata TERANG jika huruf N dan G tidak pernah terpisah? Latihan 9 (AIME 1991) Diketahui sebuah bilangan rasional, tuliskan sebagai perbandingan paling sederhana, kemudian kalikan penyebut dengan pembilang. Tentukan berhapa banyak bilangan rasional yang nilainya di antara 0 dan 1 dimana 20! menjadi hasil kalinya? Latihan 10 Seekor laba-laba mempunyai sebuah kaos kaki dan sebuah sepatu untuk masing-masing dari delapan kakinya. Tentukan banyaknya kemungkinan urutan laba-laba bisa menggunakan kaos kaki dan sepatu, dengan asumsi bahwa, untuk setiap kaki, kaos kaki harus digunakan sebelum sepatu? Latihan 11 Ada berapa banyak carakah 8 orang bisa ditempatkan dalam sebuah kursi panjang, jika:

a. tidak ada batasan dalam pengaturan posisi duduk b. orang X dan Y harus duduk berdampingan c. ada 4 pria dan 4 wanita dimana tidak ada 2 pria atau 2 wanita yang duduk

berdampingan d. ada 4 pasang suami-istri di mana setiap pasang suami-istri harus duduk berdampingan e. ada 4 pria dan mereka harus duduk berdekatan.

1.3 Permutasi dengan Pengulangan

Sekarang kita akan membahas permutasi di mana sebuah elemen/obyek dapat dipilih lebih dari satu kali. Contoh 12. Ada berapa banyak carakah huruf-huruf dari kata LALALILILU dapat

dipermutasi?

Jawab: Misalkan kita memberi index pada setiap huruf yang berulang sehingga menjadi

L1A1L2A2L3I1L4I2L5U.

Maka sekarang terdapat 10 obyek, yang dapat dipermutasi dengan 10! cara yang berbeda. Untuk setiap 10! permutasi ini, huruf A1A2 dapat dipermutasi dengan 2! cara, L1L2L3L4L5

dapat dipermutasi dengan 5! cara, dan huruf I1I2 dapat dipermutasi dengan 2! cara. Jadi sebenarnya 10! terlalu banyak terhitung, dan dapat diperbaiki menjadi 10!2! 5! 2!

Teorema Misalkan terdapat k tipe obyek, dengan: n1 obyek bertipe 1, n2 obyek bertipe 2, dst. Maka banyaknya cara penyusunan dari n1+n2+. . .+nk obyek ini adalah dengan ! ! … !

Contoh 13. Ada berapa banyak cara huruf-huruf dari kata SUMSUM bisa disusun sedemikian hingga tidak ada dua huruf yang sama berdekatan?

Page 66: Materi-OSN-MatematikaSMA

63

Jawab: Misalkan kita telah memilih SU pada dua posisi pertama, maka susunan yang mungkin bagi huruf M adalah sebagai berikut:

S U M M

S U M M

S U M M Pada kasus yang pertama, terdapat 2! =2 kemungkinan penyusunan huruf S dan U, pada kasus kedua ada 2!=2 kemungkinan, sedangkan pada kemungkinan ketiga hanya ada 1 kemungkinan. Jadi jika kata yang disusun dimulai dengan huruf S dan U, maka memunculkan 2+2+1=5 kemungkinan penyusunan. Secara umum, kita bisa memilih 3 huruf untuk posisi pertama, dan 2 huruf untuk posisi kedua. Sehingga banyaknya kemungkinan yang dicari adalah 3 ⋅ 2 ⋅ 5 =30. Latihan 14. Ada berapa banyak bilangan yang bisa dibentuk oleh angka 1, 2, 3, 4, 3, 2, 1 sehingga angka ganjil menempati posisi ganjil?

1.4 Kombinasi tanpa Pengulangan Misalkan diberikan sebuah himpunan dengan n elemen. Pemilihan k anggota himpunan tanpa memperhatikan urutan disebut k-kombinasi, dengan 0 ≤ k ≤ n. Jadi k-kombinasi adalah sebuah subhimpunan dengan k anggota (k-subhimpunan). Banyaknya cara untuk memilih r-kombinasi dari sebuah himpunan dengan n anggota dinotasikan dengan . Dengan demikian menyatakan banyaknya k-subhimpunan dari

himpunan dengan n anggota. Dapat ditunjukkan bahwa !! !

Catatan: Perhatikan bahwa 0 1 sedangkan 1 1 .

Karena n - (n - k) = k , maka untuk untuk bilangan bulat n, k berlaku sifat identitas simetri, yakni !! ! ! ! !

Hal ini bisa diinterpretasikan sebagai berikut: jika ada n pinsil warna yang berbeda, maka banyaknya cara memilih k pinsil sama dengan banyaknya cara memilih n-k pinsil yang tidak diambil.

Contoh15. 119 112 55

Page 67: Materi-OSN-MatematikaSMA

64

125 127 792.

Contoh 16. 2-kombinasi dari himpunan {P, Q, R, S} adalah

PQ, PR, PS, QR, QS, RS

Contoh 17. 3-kombinasi dari himpunan {P, Q, R, S} adalah

PQR, PQS, PRS, QRS

Contoh 18. Dari 10 buku tulis, kita bisa memilih 4 dengan menggunakan 104 210 cara.

1.5 Kombinasi dengan Pengulangan

Teorema (De Moivre) Misalkan n adalah bilangan asli. Banyaknya solusi bilangan asli yang memenuhi

x1 + x2 + · · · + xr = n adalah 1 Bukti: Tuliskan n sebagai

n =1 + 1 + … + 1,

dengan n angka 1, dan n -1 notasi + . Untuk mendekomposi n menjadi penjumlahan r bilangan kita hanya perlu memilih r – 1 notasi + , yang membuktikan teorema. Contoh 19. Berapa banyak solusi bilangan bulat non-negatif untuk persamaan

x1 + x2 + x3 + x4=11

Jawab: Contoh ini bisa diselesaikan dengan secara langsung menggunakan Teorema De Moivre. Tetapi berikut ini akan diberikan ilustrasi sehingga bisa diperoleh gambaran lebih jelas. Perhatikan barisan dari 14 (11+3) bintang di bawah ini.

* * * * * * * * * * * * * *

Page 68: Materi-OSN-MatematikaSMA

65

Pilih tiga dari 14 bintang ini, dan ubah menjadi garis tegak. Dengan demikian, ketiga garis tegak ini membagi 11 bintang menjadi 4 kelompok (sebuah kelompok dimungkinkan untuk tidak memiliki anggota). Setiap pilihan 3 garis tegak berpadanan tepat dengan satu solusi persamaan di atas, dengan memandang banyaknya bintang yang terletak dalam suatu kelompok sebagai nilai dari sebuah variable. Sebagai contoh, susunan di bawah ini berpadanan dengan solusi

x1=1, x2=0, x3=5, x4=5.

* | | * * * * * | * * * * *

Dengan demikian banyaknya solusi dari persamaan di atas adalah 11 4 14 1 143 1411 364 Contoh 20. Tentukan banyaknya lintasan terpendek dari A ke B.

Jawab: Setiap lintasan terpendek dari A ke B harus terdiri dari 9 langkah, di mana 4 di antaranya adalah langkah ke atas dan sisanya langkah ke kanan. Dengan demikian banyaknya lintasan terpendek adalah 94 95 126. Contoh 21. Misalkan terdapat sepuluh bola yang identik, dan keranjang yang dinomori 1, 2, …, 8. Tentukan banyaknya cara untuk mendistribusikan bola ke dalam keranjang agar setiap keranjang terisi sedikitnya 1 bola. Jawab: Banyaknya cara adalah 10 18 1 97 36. Jelaskan mengapa demikian!

B

A

Page 69: Materi-OSN-MatematikaSMA

66

2. Prinsip Sangkar Merpati (Pigeon-hole Principle) Prinsip sangkar merpati menyatakan bahwa jika terdapat n barang yang didistribusikan ke dalam n buah kotak, maka sedikitnya satu kotak akan menerima lebih dari satu barang. Untuk menghemat penulisan, pada contoh-contoh berikut prinsip ini ditulis sebagai PHP. Contoh 1. Jika terdapat 13 orang, maka sedikitnya ada dua orang merayakan ulang tahun pada bulan yang sama Contoh 2. kemudian jika rata-rata manusia memiliki dua juta rambut, maka setidaknya ada empat orang di Jakarta memiliki jumlah rambut yang sama di kepalanya. Perhatikan bahwa prinsip sangkar merpati bermanfaat dalam membuktikan masalah eksistensi, yakni kita menunjukkan sesuatu ‘ada’ tanpa perlu diidentifikasi secara kongkret. Contoh 3. (Putnam 1978) Misalkan A adalah himpunan dua puluh bilangan asli yang dipilih dari deret matematika 1, 4, . . . , 100. Buktikan bahwa ada dua bilangan asli berbeda dalam A yang jumlahnya 104. Jawab: Kita partisi ketigapuluh empat anggota dari deret ini menjadi delapan belas himpunan, yakni {1}, {52}, {4, 100}, {7, 97}, {10, 94}, . . . , {49, 55}. Karena kita memilih dua puluh bilangan asli dan kita mempunyai delapan belas himpunan, maka berdasarkan PHP, terdapat dua bilangan yang merupakan anggota dari himpunan yang sama. Perhatikan bahwa jumlah keduanya adalah 104. Contoh 4. Tunjukkan bahwa di antara tujuh bilangan asli positif yang nilainya kurang atau sama dengan 126, bisa kita temukan dua diantaranya, katakan a dan b, yang memenuhi b < a ≤ 2b. Jawab: Bagi bilangan-bilangan {1, 2, 3, . . . , 126} menjadi enam himpunan berikut:

{1, 2}, {3, 4, 5, 6}, {7, 8, . . . , 13, 14}, {15, 16, . . . , 29, 30}, {31, 32, . . . , 61, 62} dan {63, 64, . . . , 126}.

Berdasarkan PHP, dua dari tujuh bilangan pasti terletak dalam salah satu himpunan, dan jelas bahwa kedua bilangan tersebut akan memenuhi ketidaksamaan yang diminta. Contoh 5. Buktikan bahwa bagaimanapun lima puluh lima bilangan dipilih dari {1, 2, . . . , 100}, pasti terdapat dua bilangan yang selisihnya 10. Jawab: Pertama-tama perhatikan bahwa jika kita memilih n + 1 bilangan dari sebarang 2n bilangan asli berurutan, maka terdapat dua bilangan yang selisihnya n. Karena jika kita memasangkan 2n bilangan asli berurutan {a + 1, a + 2, . . . , a + 2n} menjadi n pasang

Page 70: Materi-OSN-MatematikaSMA

67

{a + 1, a + n + 1}, {a + 2, a + n + 2}, . . . , {a + n, a+2n}, kemudian kita pilih n + 1 bilangan, maka berdasarkan PHP terdapat dua bilangan berada dalam kelompok yang sama. Sekarang, kelompokkan 100 bilangan menjadi:

{1, 2, . . . , 20}, {21, 22, . . . , 40}, {41, 42, . . . , 60}, {61, 62, . . . , 80} dan {81, 82, . . . , 100}.

Jika kita memilih limapuluh lima bilangan, maka berdasarkan PHP, pasti terdapat suatu kelompok dimana kita memilih sedikitnya sebelas bilangan. Dengan menerapkan lemma sebelumnya pada kelompok tersebut (yakni untuk n = 10), maka terdapat dua bilangan dengan selisih 10. Contoh 6. (AHSME 1994) Namai sebuah CD dengan label ”1”, dua CD dengan label ”2”, tiga CD dengan label ”3”, . . ., dan lima puluh CD dengan label ”50”. Kemudian letakkan 1 + 2 + 3 + . . . + 50 = 1275 CD berlabel ini ke dalam suatu kotak. Beberapa CD diambil dari kotak secara acak tanpa penggantian. Berapakah jumlah minimum CD yang harus diambil untuk menjamin diperoleh setidaknya sepuluh CD dengan label yang sama? Jawab: Jika kita mengambil semua CD yang berlabel ”1”,. . .,”9” (ada 1+2+. . .+9 = 45 CD) dan sembilan CD dari setiap CD yang berlabel ”10”, . . ., ”50”, maka kita telah mengambil 45+9 ·41 = 414 CD. Maka pengambilan CD ke-415 akan memastikan terdapat setidaknya 10 CD dengan label yang sama. Contoh 7. (IMO 1964) Tujuhbelas orang saling berkorespondensi melalui email. Setiap orang berkorespondensi dengan keenambelas orang lainnya, dan diketahui mereka hanya membahas tiga topik yang berbeda. Jika setiap pasang korespondensi hanya membahas sebuah topik, buktikan bahwa terdapat setidaknya tiga orang saling berkorespondensi dengan topik yang sama. Jawab: Pilih seorang dalam kelompok tersebut, misalkan Candra. Perhatikan bahwa ia berkorespondensi dengan enam belas orang lainnya. Berdasarkan PHP, setidaknya ada enam orang yang berkorespondensi dengan Candra dan membahas topik yang sama, sebut topik tersebut sebagai topik A. Jika di antara enam orang tersebut ada dua orang yang membahas topik A juga, maka masalah selesai. Karena itu berarti Candra dan mereka berdua saling berkorespondensi dengan topic yang sama, yakni topik A. Jika di antara enam orang tersebut tidak ada dua orang yang saling berkorespondensi dengan topik A, maka keenam orang tersebut hanya boleh saling berkorespondensi dengan dua topik lainnya, sebut sebagai topik B dan C. Pilih seorang dari enam orang ini, misalkan Budi. Berdasarkan PHP, di antara lima teman korespondensi Budi ini, pasti terdapat tiga diantaranya yang saling berkorespondensi dengan topic yang sama. Tanpa mengurangi keumuman, misalkan topik B. Jika ada dua dari tiga orang ini yang saling berkorespondensi dengan topik B juga, maka masalah selesai. Tetapi jika tidak, maka setiap dua dari tiga orang ini saling berkorespondensi dengan topik C, dan artinya mereka bertiga saling berkorespondensi dengan topik yang sama, yakni topic C. Terbukti. Contoh 8. Diketahui A suatu himpunan dengan anggota sepuluh bilangan asli yang bernilai di antara 1 dan 99. Buktikan bahwa terdapat dua subhimpunan yang irisannya bukan himpunan kosong dimana jumlah anggota-anggota kedua subhimpunan adalah sama.

Page 71: Materi-OSN-MatematikaSMA

68

Jawab: Dari suatu himpunan dengan 10 anggota, kita dapat memperoleh 210−1 = 1023 subhimpunan tidak kosong. Untuk setiap subhimpunan ini, kita tentukan jumlah dari anggota-anggotanya. Perhatikan bahwa nilai maksimal yang mungkin kita peroleh dari jumlah anggota suatu subhimpunan adalah 90 + 91 + . . . + 99 = 945 < 1023. Akibatnya, berdasarkan PHP, terdapat setidaknya dua subhimpunan yang memiliki jumlah anggota sama. Contoh 9. Diberikan 9 bilangan asli sebarang yang faktor primanya anggota dari himpunan {3, 7, 11}. Buktikan bahwa terdapat setidaknya dua bilangan sehingga hasil kalinya merupakan bilangan kuadrat. Jawab: Suatu bilangan asli merupaka,n bilangan kuadrat jika semua pangkat dari faktor primanya merupakan bilangan genap. Perhatikan bahwa semua bilangan asli tersebut dapat dinyatakan sebagai 3a7b11c. Setiap triplet (a, b, c) memiliki salah satu bentuk dari 8 pola paritas berikut: (genap,genap,genap), (genap,genap,ganjil), (genap,ganjil,genap), (genap,ganjil,ganjil), (ganjil,genap,genap), ..., (ganjil, genap, ganjil), (ganjil, ganjil, genap), (ganjil,ganjil,ganjil). Karena terdapat 9 bilangan asli, maka berdasarkan PHP, terdapat dua bilangan yang triplet pangkatnya memiliki paritas yang sama. Pilih dua bilangan ini, maka hasil kalinya merupakan bilangan kuadrat, karena jumlah dari tiap pangkat yang bersesuaian merupakan bilangan genap. LATIHAN Latihan 10. Buktikan bahwa di antara n + 1 bilangan, pasti terdapat setidaknya dua bilangan yang selisihnya habis dibagi n. Latihan 10. Tunjukkan bahwa jika lima titik sebarang semuanya pada atau di dalam suatu persegi dengan panjang sisi 1, maka terdapat sepasang titik yang jaraknya kurang atau sama dengan √2/2. Latihan 12. (Hungarian Math Olympiad, 1947) Buktikan bahwa di antara enam orang dalam suatu ruangan terdapat tiga orang yang saling kenal, atau terdapat setidaknya tiga orang yang tidak saling kenal. Latihan 13. Kita menyebut suatu himpunan ”bebas jumlah” jika tidak ada dua anggota himpunan yang jumlahnya adalah anggota dari himpunan itu juga. Berapa maksimal banyaknya anggota subhimpunan dari himpunan {1, 2, . . . , 2n − 1} yang ”bebas jumlah”? Petunjuk: Perhatikan bahwa himpunan {n, n+1,n+2,. . . , 2n-1} dengan n anggota adalah ”bebas jumlah”. Tunjukkan bahwa sebarang subhimpunan dengan n + 1 anggota tidak bebas jumlah. Latihan 14. (MMPC 1992) Misalkan huruf-huruf alfabet dituliskan secara acak.

a. Buktikan bahwa pasti terdapat empat huruf konsonan yang berurutan b. Tuliskan suatu daftar huruf alfabet dimana tidak terdapat lima huruf konsonan

berurutan. c. Buktikan bahwa jika semua huruf diatur secara melingkar, maka terdapat lima huruf

konsonan berurutan.

Latihan 15. Misalkan M adalah bilangan asli tujuh belas angka dan misalkan N adalah bilangan yang diperoleh dari M dengan menuliskan angka-angka yang sama tapi dengan

Page 72: Materi-OSN-MatematikaSMA

69

urutan terbalik. Buktikan bahwa terdapat setidaknya satu angka dari representasi bilangan M +N yang genap. Latihan 16. Tidak peduli bagaimanapun lima puluh lima bilangan asli dipilih dari {1, 2, . . . , 100}, buktikan bahwa dapat dipilih dua bilangan yang memiliki selisih 9, dua yang memiliki selisih 10, dua yang memiliki selisih 12, dan 2 yang memiliki selisih 13, tetapi tidak harus ada dua yang memiliki selisih 11. Latihan 17. Diketahui mn + 1 bilangan real yang berbeda. Buktikan bahwa terdapat suatu barisan meningkat dengan setidaknya n + 1 anggota, atau barisan menurun dengan setidaknya m + 1 anggota. Latihan 18. Jika titik-titik pada suatu bidang diwarnai dengan tiga warna, tunjukkan bahwa akan selalu terdapat dua titik dengan warna yang sama berjarak satu satuan. Latihan 19. Tunjukkan bahwa jika titik-titik ada suatu bidang diwarnai dengan dua warna, maka akan selalu terdapat sebuah segitiga sama sisi yang titik sudut-titik sudutnya memiliki warna yang sama. 3. Paritas Contoh 1. Misalkan dua persegi di sudut yang berseberangan dari sebuah papan catur dibuang. Tunjukkan bahwa tidak mungkin 62 persegi yang tersisa ditutup oleh 31 domino. Jawab: Setiap domino menutup tepat satu kotak putih dan satu kotak hitam. Tetapi dua kotak persegi yang dibuang berwarna sama, sehingga jelas bahwa tidak mungkin persegi yang tersisa ditutup dengan 31 domino. Contoh 2. Bilangan 1, 2, …, 10 dituliskan dalam suatu baris. Tunjukkan bahwa bagaimanapun tanda penjumlahan atau pengurangan digunakan di depan masing-masing bilangan, maka jumlahnya tidak akan pernah menjadi 0. Jawab: Jumlah dari 1 + 2 + … + 10 = 55, suatu bilangan ganjil. Karena paritas tidak dipengaruhi oleh pemilihan tanda penjumlahan atau pengurangan, maka bagaimanpun tanda di depan masing-masing angka 1, 2, …, 10 dipilih, jumlahnya tidak mungkin menjadi 0. Definisi. Titik latis (m, n) pada suatu bidang adalah titik dengan koordinat bilangan bulat. Definisi. Titik tengah dari suatu ruas garis yang menghubungkan (x,y) ke (x1,y1) adalah titik 2 , 2

Contoh3 . Lima titik latis dipilih secara acak. Buktikan bahwa kita selalu bisa memilih dua titik sehingga titik tengah yang menghubungkan keduanya juga merupakan titik latis. Jawab: Perhatikan bahwa terdapat empat pola paritas: (genap, genap), (genap, ganjil), (ganjil, ganjil), (ganjil, genap). Berdasarkan PHP di antara lima titik latis pasti terdapat dua

Page 73: Materi-OSN-MatematikaSMA

70

titik yang memiliki paritas yang sama. Pilih dua, jelas bahwa titik tengahnya merupakan bilangan bulat juga. Untuk contoh-contoh berikut, kita perlu mengetahui nama-nama dari tetromino berikut:

Contoh 4. Budi mengambil setiap potong dari masing-masing tetromino pada gambar di atas. Tunjukkan bahwa bagaimanapun penyusunan dilakukan, tidak mungkin dihasilkan sebuah persegi panjang. Jawab: Jika penyusunan sebuah persegi panjang dimungkinkan, maka akan terdiri dari 20 persegi. Warnai persegi-persegi ini sebagaimana pewarnaan pada papan catur. Maka terdapat 10 persegi berwarna hitam dan 10 berwarna putih. Perhatikan bahwa T-tetromino selalu menutupi persegi berwarna hitam dengan jumlah ganjil, sedangkan tetromino lainnya selalu menutupi dengan jumlah genap. Sehingga banyaknya persegi warna hitam yang tertutupi adalah ganjil. Kontradiksi. Contoh 5. Tunjukkan bahwa sebuah papan catur 8×8 tidak bisa ditutupi dengan 15 straight-tetromino dan sebuah L-tetromino. Jawab: Warnai baris 1, 3, 5, 7 dengan warna hitam dan baris 2, 4, 6, 8 dengan warna putih. Straight-tetromino akan selalu menutupi persegi putih dengan jumlah genap, sedangkan L-tetromino akan selalu menutupi dengan jumlah ganjil. Jika dimungkinkan ada pengubinan yang diminta, maka kita akan menutupi ubin warna putih dengan jumlah ganjil, kontradiksi. Latihan 6. Dua puluh anak laki-laki dan perempuan duduk di sekeliling meja bundar. Tunjukkan bahwa pasti terdapat seseorang yang diapit oleh dua orang anak perempuan. Latihan 7. Angka 1, 2, …, 2001 dituliskan pada papan. Seseorang menghapuskan sepasang bilangan pada papan kemudian menuliskan selisihnya di papan. Hal tersebut dilakukan berulang kali hingga tinggal tersisa sebuah bilangan. Apakah mungkin bilangan tersebut 0? Latihan 8. Tunjukkan bahwa sebuah papan catur berukuran 10×10 tidak bisa ditutupi dengan 25 straight tetromino. Latihan 9. Tunjukkan bahwa sebuah papan catur berukuran 8×8 tidak bisa ditutupi dengan 15 T-tetromino dan sebuah tetromino. Latihan 10. Sebuah pertemuan dihadiri oleh n peserta. Sejumlah peserta saling berjabat tangan. Tida ada yang berjabat tangan dengan dirinya sendiri dan setiap dua peserta berjabat tangan paling banyak satu kali. Seseorang peserta dikatakan ganjil jika banyaknya jabat tangan yang dilakukannya adalah ganjil. Tunjukkan bahwa banyaknya peserta ganjil adalah genap.

Page 74: Materi-OSN-MatematikaSMA

71

Prinsip ini digunakan untuk menentukan kardinalitas dari gabungan himpunan-himpunan yang tidak harus saling lepas. Untuk kasus di mana banyaknya himpunan adalah dua atau tiga, dengan menggunakan diagram Venn kita dapat dengan mudah menunjukkan bahwa

|A | | | | | | |,

dan | | | | | | | | | | | | | | | |.

Secara umum, jika diberikan n buah himpunan maka kardinalitas dari gabungan n buah himpunan ini diberikan oleh | … || | 1

Contoh 1. Pada sebuah klub olahraga 10 orang menyukai tenis, 15 orang menyukai squash, 12 orang menyukai badminton. Di antara mereka, 5 orang menyukai tenis dan squash, 4 orang menyukai tenis dan badminton, 3 orang menyukai squash dan badminton, dan 2 orang menyukai ketiga olahraga ini. Berapa banyak anggota klub yang menyukai sedikitnya satu dari ketiga cabang olahraga ini? Jawab: Misalkan T, S, B, secara berturut-turut, adalah himpunan anggota klub yang menyukai tenis, squash dan badminton. Maka | | | | | | | | | | | | | | | | = 10 + 15 + 12 – 5 – 4 – 3 + 2

= 27 .